Re: [obm-l] Descobrir formula geral e provar f(n+1)=2f(n) +3

2011-05-09 Por tôpico fabio henrique teixeira de souza
Considere a(n) uma solução de f(n+1) = 2f(n)
Há infinitas soluções para tal, mas a(n) sempre será uma PG de razão 2.
Assim, uma solução é a(n) = 1.2^(n-1)

Vamos promover a mudança de variável f(n) = g(n).a(n)

Assim,
f(n+1) = 2f(n) + 3 se transforma em

g(n+1).a(n+1) = 2.g(n).a(n) + 3
g(n+1).2^n = 2.g(n).2^(n-1) + 3
g(n+1).2^n = g(n).2^n + 3

 g(n+1) = g(n) + 3.2^(-n)

Portanto,

g(1) = g(0) + 3.1
 g(2) = g(1) + 3.(1/2)
 g(3) = g(2) + 3.(1/4)
...
g(n) = g(n-1) + 3.(1/2^(n-1))

Somando:

g(n) = g(0) + 3.(1 + 1/2 + 1/4 + ... + 1/2^(n-1))
g(n) = g(0) + 3.[1/2^n - 1]/[1/2 - 1]
g(n) = g(0) - 6.[1/2^n - 1]

Note que f(0) = g(0).a(0) ==> 0 = g(0).2^(-1) ==> g(0) = 0

g(n) = 6.[1 - 1/2^n]

g(n) = 6.[2^n - 1]/2^n

f(n) = a(n).g(n) = 2^(n-1) . 6.[2^n - 1]/2^n
f(n) = 6.[2^n - 1]/2
f(n) = 3.[2^n - 1]








































































































































































































































Em 6 de maio de 2011 12:43, Julio Teixeira  escreveu:

> Pessoal, a um tempo acho que vi essa questao aki  e por acaso, ontem me
> deparei com ela em alguns foruns, e o pessoal estava com dificuldades..entao
> vou por aki a minha resolucao..
>
> questao 157 do Vol. 1 da colecao do G. Iezzi - Fundamentos de matematica
> elemtentar
>
> 157 - Seja f uma funcao, definida no conjunto dos numeros naturais, tal
> que, f(n+1)=2f(n) +3
> com f(0) = 0. Achar a formula geral de f(n) e prova-la por inducao..
>
> equacao: f(n+1)=2f(n) + 3  e f(0)=0
>
> para..
> n=0 => f(0+1)=2f(0)+3 => f(1)=3
> n=1 => f(1+1)=2f(1)+3 => f(2)=9
> n=2 => f(2+1)=2f(2)+3 => f(3)=21
> n=3 => f(3+1)=2f(3)+3 => f(4)=45
> n=4 => f(4+1)=2f(4)+3 => f(5)=93
>
> observando os valores retornado pelas imagens e pondo em produto de um
> fator por 3..
> f(1)=3  => f(1)=3*1
> f(2)=9  => f(2)=3*3
> f(3)=21 => f(3)=3*7
> f(4)=45 => f(4)=3*15
> f(5)=93 => f(5)=3*31
>
> agora observando os segundos fatores dos produtos acima nas imagens...
> comecamos com 1, depois 3, depois 7, e
>
> assim temos:
> a diferenca entre  3 e 1 = 2
> a diferenca entre  7 e 3 = 4
> a diferenca entre 15 e 7 = 8
> a diferenca entre 31 e 15 = 16
>
> obrservando essas diferencas, nota-se que temos uma PG, de razao 2, e com o
> primeiro termo sendo igual a 1
>
> assim a formula ja comeca a ficar evidente.. sendo 3 vezes essas
> diferencas...
>
> agora se montarmos essa PG, teremos..
>
> a1 = 1
> a2 = 2
> a3 = 4
> a4 = 8
> a5 = 16
>
> opa.. entao a proxima observacao a ser feita eh que, com os resultados
> obtidos temos que,por exemplo,
> f(1)=3*( a1 de nossa PG)
> f(2)=3*( a soma de a1 com o a2 de nossa PG)
> f(3)=3*( a soma de a1 com o a2  e a3 de nossa PG)
> f(4)=3*( a soma de a1 com o a2  e a3 e a4 de nossa PG)
>
> agora a formula do somatorio de nossa PG seria:
> Sn = a1 * (q^n - 1)/ (q - 1)
>
> onde substituindo, obteriamos:
> 2^n -1
>
> agora deduzimos entao que a formula geral seria: f(n)= 3 * ( 2^n - 1)
>
> para provarmos por inducao, vamos provar que eh valido para n=1
> f(1) = 3 * ( 2^1 -1)
> f(1) = 3 * ( 1 ) => f(1) = 3   ( OK, provamos para n=1 )
>
> agora substituimos por n, por um k, qualquer e obtemos:
> f(k)= 3 * (2^k -1)
>
> agora substituimos por k+1
> f(k+1)= 3 * (2^(k+1) -1)
>
> ok, agora note que se pegarmos a formula inicial e aplicarmos n=k,
> obteremos o seguinte..
> f(k+1)=2 * f(k) + 3
>
> ja que obtemos f(k+1) de nossa formula e f(k+1) da formula original, para
> provarmos que descobrimos a formula geral
> entao o resultado de f(k+1), tem que ser igual, assim tb testamos se eh
> valida para qualquer elemento, provando isso para qualquer sucessor de k, ou
> seja (k+1)
> entao temos o seguinte..
> f(k)= 3 * (2^k -1)
> f(k+1)= 3 * (2^(k+1) -1)
> f(k+1)=2 * f(k) + 3
>
> agora igualando os f(k+1), obtemos..
> 2 * f(k) + 3 = 3 * (2^(k+1) -1)
> substituindo f(k), pelo valor conhecido tb.. ( da nossa formula geral )
> 2 * (3 * (2^k -1)) + 3 = 3 * (2^(k+1) -1)
> 6 * (2^k -1) + 3 = 3 * (2^(k+1)) -3
> agora, dividimos amobs os lados por 3
> 2 * (2^k -1) + 1 = 2^(k+1) - 1
> 2^(k+1) -2 + 1 = 2^(k+1) - 1
> 2^(k+1) - 1 = 2^(k+1) - 1(OK)
> obtemos assim, a nossa prova...
>
>
>
>
>


[obm-l] Re: [obm-l] Somatório

2011-05-09 Por tôpico fabio henrique teixeira de souza
Note que i(i+1) = 2.[Combinação de i+1 escolhidos 2 a 2]

Em seguida, use uma das propriedades do Triângulo de Pascal-Tartaglia.

Em 9 de maio de 2011 14:17, Kleber Bastos  escreveu:

> Olá Pessoal,
>
> Não esotu conseguindo fazer o seguinte exercício:
>
> Provar que somatório de i=1 a n de i(i+1)  é igual a [n(n+1)(n+2)]/3
> Alguém póderia ajudar?
>
> Abraços,
>
> --
> Bastos
>


Re: [obm-l] Fwd: Identidade de Euler (OFFTOPIC)

2011-05-02 Por tôpico fabio henrique teixeira de souza
Obrigado a todos pela ajuda.

Em 28 de abril de 2011 19:44, Carlos Nehab  escreveu:

> Oi, querido amigo.
> Grande abraço
> Nehab
>
> Em 28/4/2011 17:40, Carlos Victor escreveu:
>
>> Oi  Mestre  Nehab ,
>> Gostei da sugestão e mais ainda  das n pessoas que moram em Nilópolis (
>> minha  terrinha).
>>
>> Abraços
>>
>> Carlos  Victor
>>
>> Em 28 de abril de 2011 17:21, Carlos Nehab
>>  escreveu:
>>
>> Oi, Fábio,
>>>
>>> Não resisti:
>>>
>>> Resolva os seguinte problema de duas maneiras (uma técnica básica e útil
>>> para resolver identidades deste tipo).
>>> De quantas maneira posso formar comissões de p pessoas, a partir de um
>>> total de m + n pessoas, sendo m o total de pessoas que moram no Maracanã
>>> e n
>>> as pessoas que moram em Nilópolis?
>>>
>>> Abraços,
>>> Nehab
>>>
>>> Em 28/4/2011 13:24, fabio henrique teixeira de souza escreveu:
>>>
>>> -- Mensagem encaminhada --
>>>
>>>> De: fabio henrique teixeira de souza
>>>> Data: 28 de abril de 2011 08:52
>>>> Assunto: Identidade de Euler
>>>> Para: obm-l@mat.puc-rio.br
>>>>
>>>>
>>>> Pessoal, estou batendo cabeça e não consigo demonstrar que
>>>> C(m,0).C(n,p) + C(m,1).C(n,p-1) + C(m,2).C(n,p-2) + ... + C(m,p).C(n,0)
>>>> =
>>>> C(m+n,p)
>>>>
>>>> Alguém pode me dar uma dica?
>>>>
>>>>
>>>>
>>> =
>>> Instruções para entrar na lista, sair da lista e usar a lista em
>>> http://www.mat.puc-rio.br/~obmlistas/obm-l.html
>>> =
>>>
>>>
> =
> Instruções para entrar na lista, sair da lista e usar a lista em
> http://www.mat.puc-rio.br/~obmlistas/obm-l.html
> =
>


[obm-l] Identidade de Euler

2011-05-01 Por tôpico fabio henrique teixeira de souza
Pessoal, estou batendo cabeça e não consigo demonstrar que
C(m,0).C(n,p) + C(m,1).C(n,p-1) + C(m,2).C(n,p-2) + ... + C(m,p).C(n,0) =
C(m+n,p)

Alguém pode me dar uma dica?


[obm-l] Fwd: Identidade de Euler

2011-04-28 Por tôpico fabio henrique teixeira de souza
-- Mensagem encaminhada --
De: fabio henrique teixeira de souza 
Data: 28 de abril de 2011 08:52
Assunto: Identidade de Euler
Para: obm-l@mat.puc-rio.br


Pessoal, estou batendo cabeça e não consigo demonstrar que
C(m,0).C(n,p) + C(m,1).C(n,p-1) + C(m,2).C(n,p-2) + ... + C(m,p).C(n,0) =
C(m+n,p)

Alguém pode me dar uma dica?


[obm-l] Re: [obm-l] Re: [obm-l] FRAÇÕES - conceito

2011-03-24 Por tôpico fabio henrique teixeira de souza
Ralph, obrigado.
Além de aprender com você, ainda me divirto.

EMMOSC (em minha modesta opinião sobre convenções):
- fração é exatamente o que diz a SMO;
- 0 é natural;
- futebol com jogadores de madeira é "totó";
- a fruta é "tangerina"

Mas não, não vou encarar.
Até porque você é maior, mais velho e mais inteligente do que eu.

Forte abraço.

Fabio Henriqu


Em 23 de março de 2011 18:18, Ralph Teixeira  escreveu:

> Minha resposta é "diplomática" -- depende do que você chamar de
> fração. Defina do seu jeito, que seja conveniente para o que você quer
> fazer, e deixe claro a todos o que você está fazendo. Depois, seja
> coerente.
>
> (Ou seja, enrolei enrolei e não respondi.)
>
> Em Minha Modestíssima Opinião, fração é qualquer expressão do tipo a/b
> onde a e b são números ou até mesmo outras expressões. Então 1/(raiz
> de 2) é uma fração tanto quando 7/1 ou 25/pi ou (x+cos(y))/(z+w^2). Eu
> também diria que 3 não é uma fração, mas pode ser escrito como 3/1,
> que é uma fração... para mim, 45.78 não é fração, mas PODE SER ESCRITO
> como uma fração, 4578/100.
>
> Mas isso tudo é EMMO... Não, minto, é EMMC (Em Minha Modestíssima
> Convenção). Poxa, EMMC, 0 é natural, 0^0=1, aquele futebol com
> jogadores de madeira é "pebolim" e aquela fruta é "mixirica" Não
> gostou? Vai encarar? :) :) :) :)
>
> Abraço,
> Ralph
>
> 2011/3/21 fabio henrique teixeira de souza :
> > Senhores, 1/(raiz de 2) é uma fração?
>
> =
> Instruções para entrar na lista, sair da lista e usar a lista em
> http://www.mat.puc-rio.br/~obmlistas/obm-l.html
> =
>


[obm-l] FRAÇÕES - conceito

2011-03-21 Por tôpico fabio henrique teixeira de souza
Senhores, 1/(raiz de 2) é uma fração?


[obm-l] Concurso Pré Vestibular CEDERJ

2010-01-27 Por tôpico fabio henrique teixeira de souza
Peço aos amigos que divulguem no Estado do Rio de Janeiro:



Concurso para seleção de tutores do Pré Vestibular do CEDERJ.



Inscrições on-line até 31/01 para graduados e graduandos não só de
matemática mas de todas as disciplinas.



Informações em http://www.pvs.cederj.edu.br/professores/


Um abraço e obrigado.
Fabio Henrique


[obm-l] Re: [obm-l] Re: [obm-l] Re: [obm-l] Re: [obm-l] Ques tão de Probabilidade - CESPE

2009-01-24 Por tôpico fabio henrique teixeira de souza
Valeu. Um abraço.

2009/1/24 Marcelo Salhab Brogliato 

> Olá Fábio,
> eita, realmente não são equiprováveis. Desculpe pela falha.. hehehe
> Acho que é só multiplicar as probabilidades dos casos favoráveis e somar:
> 11 .. neste caso, temos: 1/2*1/2 = 1/4
> 1011 .. neste caso, temos: 1/2*1/2*1/2*1/2 = 1/16
>
> logo, P = 1/4 + 1/16 = 5/16
>
> espero nao ter errado novamente ;) hehe
> mas desta vez bateu com sua resposta... acho que acertamos!! :D
>
> abraços,
> Salhab
>
>
> 2009/1/23 fabio henrique teixeira de souza 
>
>  Salhab, agradeço a ajuda. No entanto, acho que vc cometeu um engano. Não
>> podemos calcular a chance de A ser campeão por 2 casos favoráveis em um
>> total de 5, posto que os 5 não são equiprováveis. Estou errado?
>> Abraço.
>>
>> 2009/1/23 Marcelo Salhab Brogliato 
>>
>> Olá Fábio,
>>> dado que A perdeu o primeiro jogo, temos as seguinte possiveis
>>> configuracoes para os demais jogos:
>>> vamos convencionar: 1 = A venceu, 0 = B venceu (ou A perdeu).
>>> 0 (neste caso B é campeao)
>>> 11 (neste caso A é campeao)
>>> 100 (neste caso B é campeao)
>>> 1010 (neste caso B é campeao)
>>> 1011 (neste caso A é campeao)
>>> logo, temos 2/5 = 40% de chance de A ser campeão.
>>>
>>> Outra maneira seria calcular: P(A ser campeao | A perdeu a primeira
>>> partida) = P(A ser campeao e A perder a primeira partida)/P(A perder a
>>> primeira partida)
>>>
>>> P(A perder a primeira partida) = 1/2
>>> P(A ser campeao e A perder a primeira partida) = 2/10 [basta ver que
>>> temos 2 casos favoraveis em 10, pois temos que considerar os casos em que A
>>> ganhou a primeira partida, que sao analogos aos acima]
>>>
>>> assim: P = (2/10)/(1/2) = 2/5 = 40%
>>>
>>> acho que é isso!!
>>> abraços,
>>> Salhab
>>>
>>>
>>>
>>>
>>> 2009/1/22 fabio henrique teixeira de souza 
>>>
>>>  Senhores, ao fazer a questão abaixo, encontrei como probabilidade 5/16.
>>>> De saída, o problema não deixa claro que as probabilidades de vitória de A 
>>>> e
>>>> de B são iguais. Considerei-as iguais (a 1/2). No entanto, o gabarito
>>>> oficial diz que a declaração é CERTA. Gostaria que alguém também a fizesse
>>>> para que eu possa ter idéia se estou errando ou acertando. Aguardo o
>>>> retorno.
>>>>
>>>>
>>>>
>>>>
>>>> Analise em certo ou errado:
>>>>
>>>> A etapa final de um torneio de futebol será disputada entre os
>>>>
>>>> times A e B, e o campeão será o time que vencer duas partidas
>>>>
>>>> seguidas ou um total de três partidas. Considerando que os jogos
>>>>
>>>> que terminarem empatados serão decididos nos pênaltis, de forma
>>>>
>>>> que sempre haja um vencedor, julgue os itens que se seguem.
>>>>
>>>> 109
>>>> A chance de o time A ser campeão tendo perdido o primeiro
>>>>
>>>> jogo é de 20%.
>>>>
>>>
>>>
>>
>


[obm-l] Dúvida de lógica

2009-01-23 Por tôpico fabio henrique teixeira de souza
Amigos, cá venho eu com mais um pepino.

Ontem, um amigo me perguntou: - "2 é primo e 3 é primo" é uma proposição
simples ou composta?
Prontamente respondi que é composta.

A seguir, perguntou: - e a proposição "2 e 3 são números primos"? É simples
ou composta.
Desta feita, não tive certeza, apenas a convicção de que é uma proposição
simples. No entanto, gostaria de sair da convicção para a certeza e, se
possível, descobrir alguma referência bibliográfica que embase esta certeza.

A importância dessa distinçaõ se dá por conta da negação.

Se, de fato, for simples, a negação de "2 e 3 são números primos" será "2 e
3 não são números primos" e não, "2 não é primo ou 3 não é primo". Parece-me
que a chave da questão é que o e que une 2 e 3 na expressão "2 e 3 são
números primos" não é um conectivo lógico-matemático e sim uma conjunção da
língua portuguesa para construir um sujeito composto.

Aguardo as opiniões.

Um forte abraço.


Fabio Henrique TEIXEIRA de Souza (Olha aí, Ralph! Agora só vai em caixa
alta.) :)


[obm-l] Re: [obm-l] Re: [obm-l] Re: [obm-l] Re: [obm-l] Ques tão de Probabilidade - CESPE

2009-01-23 Por tôpico fabio henrique teixeira de souza
Ralph, valeu pela moral e pela resposta.

2009/1/23 Ralph Teixeira 

> Concordo com o Fábio. "Não são equiprováveis" é a pedra no sapato de
> 98% dos problemas de probabilidade que dão errado... :)
>
> Então, usando o raciocínio do Marcelo, temos as seguintes opções (dada
> a primeira derrota de A, daqui para a frente):
> >> 0 (neste caso B é campeao) com probabilidade 1/2
> >> 11 (neste caso A é campeao) com probabilidade 1/2.1/2=1/4
> >> 100 (neste caso B é campeao) com probabilidade 1/2.1/2.1/2=1/8
> >> 1010 (neste caso B é campeao) com probabilidade 1/2.1/2.1/2.1/2=1/16
> >> 1011 (neste caso A é campeao) com probabilidade 1/2.1/2.1/2.1/2=1/16
>
> Note que eu fiz as seguintes hipóteses que o enunciado não deixa
> claras, mas que creio serem as mais razoáveis possíveis:
> i) A e B têm 50% de chance cada de vencer cada partida (que o próprio
> Fábio já havia destacado que não estava explícito no enunciado, e, na
> minha opinião, devia)
> ii) Partidas distintas são independentes entre si -- ou seja, que não
> interessa se A está para ser campeão ou B está desesperado, os times
> continuam com 50% de chance cada em cada partida, independentemente da
> história passada. Por isso que eu posso usar que p(1011) é o produto
> 1/2.1/2.1/2.1/2.
>
> Então, p(A ser campeão)=p(11)+p(1011)=5/16. Tô com o Fábio (que,
> afinal, é da família Teixeira, então **não podia** estar enganado :)
> ).
>
> Abraço,
>   Ralph
>
>
> 2009/1/23 fabio henrique teixeira de souza :
>  > Salhab, agradeço a ajuda. No entanto, acho que vc cometeu um engano.
> Não
> > podemos calcular a chance de A ser campeão por 2 casos favoráveis em um
> > total de 5, posto que os 5 não são equiprováveis. Estou errado?
> > Abraço.
> >
> > 2009/1/23 Marcelo Salhab Brogliato 
> >>
> >> Olá Fábio,
> >> dado que A perdeu o primeiro jogo, temos as seguinte possiveis
> >> configuracoes para os demais jogos:
> >> vamos convencionar: 1 = A venceu, 0 = B venceu (ou A perdeu).
> >> 0 (neste caso B é campeao)
> >> 11 (neste caso A é campeao)
> >> 100 (neste caso B é campeao)
> >> 1010 (neste caso B é campeao)
> >> 1011 (neste caso A é campeao)
> >> logo, temos 2/5 = 40% de chance de A ser campeão.
> >>
> >> Outra maneira seria calcular: P(A ser campeao | A perdeu a primeira
> >> partida) = P(A ser campeao e A perder a primeira partida)/P(A perder a
> >> primeira partida)
> >>
> >> P(A perder a primeira partida) = 1/2
> >> P(A ser campeao e A perder a primeira partida) = 2/10 [basta ver que
> temos
> >> 2 casos favoraveis em 10, pois temos que considerar os casos em que A
> ganhou
> >> a primeira partida, que sao analogos aos acima]
> >>
> >> assim: P = (2/10)/(1/2) = 2/5 = 40%
> >>
> >> acho que é isso!!
> >> abraços,
> >> Salhab
> >>
> >>
> >>
> >>
> >> 2009/1/22 fabio henrique teixeira de souza 
> >>>
> >>> Senhores, ao fazer a questão abaixo, encontrei como probabilidade 5/16.
> >>> De saída, o problema não deixa claro que as probabilidades de vitória
> de A e
> >>> de B são iguais. Considerei-as iguais (a 1/2). No entanto, o gabarito
> >>> oficial diz que a declaração é CERTA. Gostaria que alguém também a
> fizesse
> >>> para que eu possa ter idéia se estou errando ou acertando. Aguardo o
> >>> retorno.
> >>>
> >>>
> >>>
> >>>
> >>> Analise em certo ou errado:
> >>>
> >>> A etapa final de um torneio de futebol será disputada entre os
> >>>
> >>> times A e B, e o campeão será o time que vencer duas partidas
> >>>
> >>> seguidas ou um total de três partidas. Considerando que os jogos
> >>>
> >>> que terminarem empatados serão decididos nos pênaltis, de forma
> >>>
> >>> que sempre haja um vencedor, julgue os itens que se seguem.
> >>>
> >>> 109
> >>>
> >>> A chance de o time A ser campeão tendo perdido o primeiro
> >>>
> >>> jogo é de 20%.
> >
> >
>
> =
> Instruções para entrar na lista, sair da lista e usar a lista em
> http://www.mat.puc-rio.br/~obmlistas/obm-l.html
> =
>


[obm-l] Re: [obm-l] Re: [obm-l] Questão de Probabilidade - CESPE

2009-01-23 Por tôpico fabio henrique teixeira de souza
Salhab, agradeço a ajuda. No entanto, acho que vc cometeu um engano. Não
podemos calcular a chance de A ser campeão por 2 casos favoráveis em um
total de 5, posto que os 5 não são equiprováveis. Estou errado?
Abraço.

2009/1/23 Marcelo Salhab Brogliato 

> Olá Fábio,
> dado que A perdeu o primeiro jogo, temos as seguinte possiveis
> configuracoes para os demais jogos:
> vamos convencionar: 1 = A venceu, 0 = B venceu (ou A perdeu).
> 0 (neste caso B é campeao)
> 11 (neste caso A é campeao)
> 100 (neste caso B é campeao)
> 1010 (neste caso B é campeao)
> 1011 (neste caso A é campeao)
> logo, temos 2/5 = 40% de chance de A ser campeão.
>
> Outra maneira seria calcular: P(A ser campeao | A perdeu a primeira
> partida) = P(A ser campeao e A perder a primeira partida)/P(A perder a
> primeira partida)
>
> P(A perder a primeira partida) = 1/2
> P(A ser campeao e A perder a primeira partida) = 2/10 [basta ver que temos
> 2 casos favoraveis em 10, pois temos que considerar os casos em que A ganhou
> a primeira partida, que sao analogos aos acima]
>
> assim: P = (2/10)/(1/2) = 2/5 = 40%
>
> acho que é isso!!
> abraços,
> Salhab
>
>
>
>
> 2009/1/22 fabio henrique teixeira de souza 
>
>  Senhores, ao fazer a questão abaixo, encontrei como probabilidade 5/16.
>> De saída, o problema não deixa claro que as probabilidades de vitória de A e
>> de B são iguais. Considerei-as iguais (a 1/2). No entanto, o gabarito
>> oficial diz que a declaração é CERTA. Gostaria que alguém também a fizesse
>> para que eu possa ter idéia se estou errando ou acertando. Aguardo o
>> retorno.
>>
>>
>>
>>
>> Analise em certo ou errado:
>>
>> A etapa final de um torneio de futebol será disputada entre os
>>
>> times A e B, e o campeão será o time que vencer duas partidas
>>
>> seguidas ou um total de três partidas. Considerando que os jogos
>>
>> que terminarem empatados serão decididos nos pênaltis, de forma
>>
>> que sempre haja um vencedor, julgue os itens que se seguem.
>>
>> 109
>> A chance de o time A ser campeão tendo perdido o primeiro
>>
>> jogo é de 20%.
>>
>
>


[obm-l] Questão de Probabilidade - CESPE

2009-01-22 Por tôpico fabio henrique teixeira de souza
 Senhores, ao fazer a questão abaixo, encontrei como probabilidade 5/16. De
saída, o problema não deixa claro que as probabilidades de vitória de A e de
B são iguais. Considerei-as iguais (a 1/2). No entanto, o gabarito oficial
diz que a declaração é CERTA. Gostaria que alguém também a fizesse para que
eu possa ter idéia se estou errando ou acertando. Aguardo o retorno.




Analise em certo ou errado:

A etapa final de um torneio de futebol será disputada entre os

times A e B, e o campeão será o time que vencer duas partidas

seguidas ou um total de três partidas. Considerando que os jogos

que terminarem empatados serão decididos nos pênaltis, de forma

que sempre haja um vencedor, julgue os itens que se seguem.

109 A chance de o time A ser campeão tendo perdido o primeiro

jogo é de 20%.


Re: [obm-l] Questão CHATA ???

2008-12-18 Por tôpico Fabio Henrique
eu pensei assim só que eu prensei em relação aos soldados. um arranjo de
distribuir 5 fuzis para 2 soldados
soldado 1 pode receber 5
soldado 2 pode receber 4

5*4=20 , mas esse raciocionio esta errado,. eu entendo pq o seu esta certo ,
mas não pq o meu está errado.

On Thu, Dec 18, 2008 at 1:57 PM, Carlos Nehab  wrote:

>  Oi, gente,
>
> Eu acho que os fusíveis (como diz meu porteiro) são indistinguíveis e a
> questão é apenas saber os quantitativos diferentes de fusis que cada soldado
> diferente (distinguíveis) pode receber.  Não acham?
>
> Nehab
>
> PS: Oi, Ponce, agora sou um cara sério. Virei vovô de um lindo menino:
> Felipe. Saudades.
>
> Rogerio Ponce escreveu:
>
> Oi Fabio,
> conhece o problema sobre quantos pratos possiveis podem ser montados
> com uma salada, de um total de 5 saladas disponiveis, uma carne de um
> total de 3 carnes, um acompanhamento de um total de 5 acompanhamentos
> , e uma sobremesa de um total de 4 sobremesas?
>
> Voce sabe que existem 5*3*5*4=300 pratos possiveis, certo?
>
> Pois o problema dos fuzis e' a mesma coisa:
> Para o primeiro fuzil , existem 2 opcoes de entrega (soldado A ou soldado B)
> Para o segundo fuzil tambem existem 2 opcoes de entrega, e assim por diante.
> Ao final, podemos distribuir os 5 fuzis de 2*2*2*2*2 = 32 formas diferentes.
>
> Entretanto, como cada soldado recebe pelo menos 1 fuzil, devemos
> eliminar a distribuicao em que o soldado A nao recebeu fuzil algum, e
> a distribuicao em que o soldado B nao recebeu fuzil algum.
> O total sera' 32 - 2 = 30 fuzis.
>
> []'s
> Rogerio Ponce
>
>
>
> 2008/12/18 Fabio Henrique  
> :
>
>
>  Rogerio , vc acertou a resposta é 30. Mas eu nao entendi o seu raciocinio.
>
> 2008/12/18 Bruno França dos Reis  
>
>  Boa Rogério, acabo de ver que cometi um erro na minha segunda solução (eu
> tinha afirmado que eram 32 formas, mas esqueci do detalhe de que cada
> soldado tinha que ter no mínimo um fuzil).
>
> --
> Bruno FRANÇA DOS REIS
>
> msn: brunoreis...@hotmail.com
> skype: brunoreis666
> tel: +33 (0)6 28 43 42 16
> http://www.brunoreis.comhttp://blog.brunoreis.com
>
> e^(pi*i)+1=0
>
>
> 2008/12/18 Rogerio Ponce  
>
>  Ola' pessoal,
> esse enunciado admite varias interpretacoes, pois os fuzis podem ser
> iguais ou diferentes entre si, e a palavra "distribuicao" pode se
> referir ao ato de distribuir (e nesse caso, se os fuzis forem
> diferentes entre si, devemos considerar a ordem em que eles sao
> entregues), ou ao resultado final do "ato de distribuir" (nesse caso,
> a ordem em que os fuzis foram entregues nao importa).
>
> Considerando fuzis diferentes , e apenas o resultado da entrega,
> teriamos a seguinte solucao, por exemplo:
> Cada fuzil tem 2 opcoes para ser entregue.
> Como sao 5 fuzis, ha' 2**5 = 32 opcoes.
> Como nao podemos ter todos os fuzis com o soldado A , ou todos com o
> soldado B, devemos eliminar 2 opcoes desse total.
> Assim, existem 30 formas de distribuicao dos fuzis.
>
> Observem que esta e' apenas uma das interpretacoes possiveis.
> []'s
> Rogerio Ponce
>
>
>
>
> 2008/12/17 Fabio Henrique  
> :
>
>
>  Essa questão é de um concurso que eu fiz e eu nao consigo entender o
> gabarito , espero que me ajudem , sem mais Fábio
>
> "Dois soldados serão designados para uma mesma missão
> e  para  eles  serão  distribuídos  (sem  sobra)  5  fuzis  de  tal
> forma  que  cada  soldado  receba  ao menos  um  fuzil. Essa
> distribuição deverá ser feita de n formas.
> Então, pode-se afirmar que n vale : "
>
> Não vou postar a reposta pois pode interferir na resolução do problema.
> Desde já Obrigado.
>
> --
> Be Free
> Use LINUX
> Linux #244712
>
>
>
>  =
> Instruções para entrar na lista, sair da lista e usar a lista 
> emhttp://www.mat.puc-rio.br/~obmlistas/obm-l.html
> =
>
>
>  --
> Be Free
> Use LINUX
> Linux #244712
>
>
>
>  =
> Instruções para entrar na lista, sair da lista e usar a lista 
> emhttp://www.mat.puc-rio.br/~obmlistas/obm-l.html
> =
>
>
>
>  =
> Instruções para entrar na lista, sair da lista e usar a lista em
> http://www.mat.puc-rio.br/~obmlistas/obm-l.html=




-- 
Be Free
Use LINUX
Linux #244712


Re: [obm-l] Questão CHATA ???

2008-12-18 Por tôpico Fabio Henrique
Rogerio , vc acertou a resposta é 30. Mas eu nao entendi o seu raciocinio.

2008/12/18 Bruno França dos Reis 

> Boa Rogério, acabo de ver que cometi um erro na minha segunda solução (eu
> tinha afirmado que eram 32 formas, mas esqueci do detalhe de que cada
> soldado tinha que ter no mínimo um fuzil).
>
> --
> Bruno FRANÇA DOS REIS
>
> msn: brunoreis...@hotmail.com
> skype: brunoreis666
> tel: +33 (0)6 28 43 42 16
>
> http://www.brunoreis.com
> http://blog.brunoreis.com
>
> e^(pi*i)+1=0
>
>
> 2008/12/18 Rogerio Ponce 
>
> Ola' pessoal,
>> esse enunciado admite varias interpretacoes, pois os fuzis podem ser
>> iguais ou diferentes entre si, e a palavra "distribuicao" pode se
>> referir ao ato de distribuir (e nesse caso, se os fuzis forem
>> diferentes entre si, devemos considerar a ordem em que eles sao
>> entregues), ou ao resultado final do "ato de distribuir" (nesse caso,
>> a ordem em que os fuzis foram entregues nao importa).
>>
>> Considerando fuzis diferentes , e apenas o resultado da entrega,
>> teriamos a seguinte solucao, por exemplo:
>> Cada fuzil tem 2 opcoes para ser entregue.
>> Como sao 5 fuzis, ha' 2**5 = 32 opcoes.
>> Como nao podemos ter todos os fuzis com o soldado A , ou todos com o
>> soldado B, devemos eliminar 2 opcoes desse total.
>> Assim, existem 30 formas de distribuicao dos fuzis.
>>
>> Observem que esta e' apenas uma das interpretacoes possiveis.
>> []'s
>> Rogerio Ponce
>>
>>
>>
>>
>> 2008/12/17 Fabio Henrique :
>> > Essa questão é de um concurso que eu fiz e eu nao consigo entender o
>> > gabarito , espero que me ajudem , sem mais Fábio
>> >
>> > "Dois soldados serão designados para uma mesma missão
>> > e  para  eles  serão  distribuídos  (sem  sobra)  5  fuzis  de  tal
>> > forma  que  cada  soldado  receba  ao menos  um  fuzil. Essa
>> > distribuição deverá ser feita de n formas.
>> > Então, pode-se afirmar que n vale : "
>> >
>> > Não vou postar a reposta pois pode interferir na resolução do problema.
>> > Desde já Obrigado.
>> >
>> > --
>> > Be Free
>> > Use LINUX
>> > Linux #244712
>> >
>>
>> =
>> Instruções para entrar na lista, sair da lista e usar a lista em
>> http://www.mat.puc-rio.br/~obmlistas/obm-l.html<http://www.mat.puc-rio.br/%7Eobmlistas/obm-l.html>
>> =
>>
>
>


-- 
Be Free
Use LINUX
Linux #244712


[obm-l] Questão CHATA ???

2008-12-17 Por tôpico Fabio Henrique
Essa questão é de um concurso que eu fiz e eu nao consigo entender o
gabarito , espero que me ajudem , sem mais Fábio

"Dois soldados serão designados para uma mesma missão
e  para  eles  serão  distribuídos  (sem  sobra)  5  fuzis  de  tal
forma  que  cada  soldado  receba  ao menos  um  fuzil. Essa
distribuição deverá ser feita de n formas.
Então, pode-se afirmar que n vale : "

Não vou postar a reposta pois pode interferir na resolução do problema.
Desde já Obrigado.

-- 
Be Free
Use LINUX
Linux #244712


Re: [obm-l] Provas do IME, versao 13

2007-11-23 Por tôpico Fabio Henrique
Qual o Link das Provas ???

On 11/23/07, Sergio Lima Netto <[EMAIL PROTECTED]> wrote:
>
> Caros colegas,
> Disponibilizei hoje a versao 13 do material
> com as provas de matematica do vetibular do IME.
> Nesta nova versao incluo as provas de 2007/2008
> e algumas pequenas correcoes.
> Abraco,
> sergio
> =
> Instruções para entrar na lista, sair da lista e usar a lista em
> http://www.mat.puc-rio.br/~obmlistas/obm-l.html
> =
>



-- 
Be Free
Use LINUX
Linux #244712


Re: [obm-l] PSSC

2007-11-22 Por tôpico fabio henrique teixeira de souza
Obrigado.

Em 18/11/07, João Luís Gomes Guimarães <[EMAIL PROTECTED]> escreveu:
>
>  Uma alternativa é procurar em sebos, você vai achar uma edição em
> português. Um ótimo site para procurar livros usados é um que congrega
> centenas de sebos por todo o país:
>
> www.estantevirtual.com.br
>
> Espero tê-lo ajudado. Um abraço,
>
> João Luís
>
> ----- Original Message -
> *From:* fabio henrique teixeira de souza <[EMAIL PROTECTED]>
> *To:* obm-l@mat.puc-rio.br
>  *Sent:* Saturday, November 17, 2007 11:11 AM
> *Subject:* [obm-l] PSSC
>
>
> Alguém sabe onde posso comprar o PSSC, livro sensacional de física.
>
> Um abraço.
>
> Fabio
>
>


[obm-l] PSSC

2007-11-17 Por tôpico fabio henrique teixeira de souza
Alguém sabe onde posso comprar o PSSC, livro sensacional de física.

Um abraço.

Fabio


Re: [obm-l] OFF TOPIC absolutamente INCONVENIENTE

2007-10-02 Por tôpico fabio henrique teixeira de souza
Já trabalhei lá. Não achei que emitir minha opinião fosse causar tamanha
confusão. De qualquer forma, peço desculpas.

Em 27/09/07, Carlos Nehab <[EMAIL PROTECTED]> escreveu:
>
> Por favor, calma, Tio Cabri,
>
> Como eu me manifestei contra este tipo de off topic, gostaria de
> esclarecer minha posição.
>
> Veja se você concorda:
>
> Indicar livros na Lista para alguém que solicita dicas sobre um
> determinado tema, me parece perfeitamente adequado;  até dizer onde o livro
> pode ser encontrado, especialmente se é um clássico difícil de achar.   Não
> vejo nada de mais.
>
> Mas acho diferente, por exemplo, como já aconteceu aqui mais de uma vez,
> mandarem um email para a lista, "do nada", dizendo que um determinado livro
> foi publicado e está à venda em algum lugar.  Você não concorda?  Haveria,
> subliminarmente, uma ação de marketing através da Lista, que não é adequado.
>
> Assim, penso que da mesma maneira, indicar Colégios, Cursos, etc, por
> melhor que seja sua causa (ajudar um amigo, como você disse) gera uma
> situação desconfortável na lista, pois muitos e muitos participantes já
> cursaram tais instituições, ou as cursam, ou até trabalham nas mesmas.
> Assim, mesmo uma pergunta aparentemente banal como a sua, gera uma situação
> quase de enquete e não é adequada.
>
> Veja, você mesmo foi testemunha, recentemente, de um email enviado por
> mim, infelizmente inadequado, por ter deixado margem a interpretações
> igualmente inadequadas.Como diz meu filho, sempre atento a NetEtiqueta
> (acredite, existe isto!)  pisei na bola.  E pedi desculpas.
>
> Uma maneira de resolver questões como a que você propôs seria talvez
> escrever em off para pessoas da lista com quem você mais se identifica e
> fazer sua "enquete" fora da Lista.  Eu já fiz isto dezenas de vezes e também
> já recebi dezenas de emails por "fora"...
>
> Finalizo apenas reafirmando que discordância não significa falta de
> respeito.   Significa apenas, entre pessoas de bem, que somos diferentes e
> pensamos diferente.
>
> Um grande abraço,
> Nehab
>
>
>
> Tio Cabri st escreveu:
>
> Tem alguém nessa lista que está de saca... e esse não sou 
> eu.itamentepertinente;
>  O uso do off topic não pode ser direcionado a alguém como deseja o Sr
> Anselmo.
> O uso do off topic é algo fora da lista mas que muitos daqueles que usam a
> lista podem responder.
> E vou falar outra coisa:  vocês estão com o testosterona a mil, perguntei
> uma coisa importante para mim e sei que muitos dessa lista podem ajudar.
> Mas tem gente que gosta de criticar, acha bonito saca... os outros, vou
> fazer o quê?
> Quando eu morava no rio em 1970 poderia dizer que o curso impacto era um
> bom curso preparatório para ime ita.
> Hoje eu desconheço, e por isso perguntei na lista.
> Ora qual o problema disso?
> Muitos às vezes perguntam um bom livro e cada um dá a sua opinião e não me
> lembro de alguém sair em defesa dos autores desses livros citados.
> Fala sério, chega desse assunto, vamos voltar para o trabalho e para os
> estudos que é o fim dessa lista e àqueles que detestam o off topic
> simplesmente é só não ler.
> Abraços
> Cabri
>
> - Original Message -
> *From:* Anselmo Sousa <[EMAIL PROTECTED]>
> *To:* obm-l@mat.puc-rio.br
> *Sent:* Wednesday, September 26, 2007 10:26 PM
> *Subject:* RE: [obm-l] OFF TOPIC absolutamente INCONVENIENTE
>
> ACHO QUE ESSE NEGÓCIO DE OFF TOPIC CHEGOU A UM LIMITE...
>
> PODERÍAMOS PARAR, POR GENTILEZ, COM ESSA PRAGA...
>
> TODOS SABEMOS E-MAILS UNS DOS OUTROS...
>
> OFF TOPIC DEVE SER MESMO OFF TOPIC E, POR ISSO MESMO, NÃO DEVE APARECER
> AQUI...
>
>
> NÃO AO OF TOPIC!!!
>
>
>
>
>  --
> Date: Wed, 26 Sep 2007 21:39:14 -0300
> From: [EMAIL PROTECTED]
> To: obm-l@mat.puc-rio.br
> Subject: [obm-l] OFF TOPIC absolutamente INCONVENIENTE
>
> Oi, gente,
>
> Com TODA franqueza, eu acho que por melhor que sejam as intenções de
> atender a algum amigo, este tipo de informação é mais do que OFF TOPIC:  é
> absolutamente INCONVENIENTE e sem qualquer cabimento.
>
> Há nesta lista, naturalmente,  inúmeros profissionais sérios que trabalham
> nestas instituições e, nestas condições, solicitações desta natreza sequer
> deveriam  ser formuladas.
>
> Nehab
>
>
> fabio henrique teixeira de souza escreveu:
>
> Ponto de Ensino
>
> Em 20/09/07, *Tio Cabri st* <[EMAIL PROTECTED]> escreveu:
>
> Bom dia, preciso indicar a um amigo o melhor curso preparatório para o
> concurso do IME - ITA na cidade do Rio de Janeiro.
> Gostaria da opinião dos senhores dessa lista.
> Obrigado
> Cabri
>
> ===

Re: [obm-l] off topic: cursinho preparatorio

2007-09-26 Por tôpico fabio henrique teixeira de souza
Ponto de Ensino

Em 20/09/07, Tio Cabri st <[EMAIL PROTECTED]> escreveu:
>
> Bom dia, preciso indicar a um amigo o melhor curso preparatório para o
> concurso do IME - ITA na cidade do Rio de Janeiro.
> Gostaria da opinião dos senhores dessa lista.
> Obrigado
> Cabri
>
> =
> Instruções para entrar na lista, sair da lista e usar a lista em
> http://www.mat.puc-rio.br/~nicolau/olimp/obm-l.html
> =
>


[obm-l] Re:

2007-02-03 Por tôpico Fabio Henrique

EMULE , BITTORRENT

On 2/3/07, André Smaira <[EMAIL PROTECTED]> wrote:


Vcs poderiam me informar onde acho o Maple mais recente possivel e de
graça para baixar?
Eu tinha o original mas fiz uma modificações no meu pc e nao consegui
instalar +!!!

__
Fale com seus amigos de graça com o novo Yahoo! Messenger
http://br.messenger.yahoo.com/





--
Be Free
Use LINUX
Linux #244712


Re: [obm-l] Motivos para votar em LULA!!!

2006-10-23 Por tôpico Fabio Henrique
Primeiro sou Eleitor do Lula Segundo voce está enganado meu caro. Caso o presidente eleito sofra um processo de IMpeachmente antes dos dois anos de mandato , será feita nova eleição. Porém IMpeachemnt é um processo politico  e com 62% da populção apoiando o Presidente , acho díficil , esperem mais 4 anos e tente a sorte com o Aecio. Lula de Novo , com a força do POvo.

On 10/22/06, Vitor Tomita Silva <[EMAIL PROTECTED]> wrote:
Olhem agora o nível dos eleitores do Alckmin, generalizando demais.Claro, o que o sr. eleitor do Lula fez foi bem pior... ainda mais mandando
propaganda numa lista de matemática, sendo que a imensa maioria da galera deexatas odeia o Lula. Hugo, falta-lhe raciocínio lógico-matemático.Acho que o que falta nessa eleição não é só candidato, é eleitor bonzinho
também. Se fôssemos todos fofos e discutíssemos MATEMÁTICA nessa lista,seria bem melhor. Afinal, todos sabemos que, com a OAB já dizendo quederruba o Lula se ele for reeleito, o Alckmin (provavelmente) assume até se
perder.>From: "Fernando A Candeias" <[EMAIL PROTECTED]>>Reply-To: obm-l@mat.puc-rio.br>To: 
obm-l@mat.puc-rio.br>Subject: Re: [obm-l] Motivos para votar em LULA!!!>Date: Sun, 22 Oct 2006 11:02:25 -0200>>*Acho que é uma atitude que reflete bem o nível dos eleitores do Lula: vale
>tudo>>* Em 22/10/06, Hugo Leonardo da Silva Belisário><[EMAIL PROTECTED]>>escreveu:Me limito a citar vário links nos quais fundamento meu voto em LULA para
>>presidente. Leiam,http://carosamigos.terra.com.br/da_revista/edicoes/ed114/valeapena.asp>>
http://forums.ecomm.com.br/cgi/dnewsweb.exe?cmd=article&group=forum.carosamigos&item=7215&utag=
>>http://forums.ecomm.com.br/cgi/dnewsweb.exe?cmd=article&group=forum.carosamigos&item=7241&utag=
>>http://forums.ecomm.com.br/cgi/dnewsweb.exe?cmd=article&group=forum.carosamigos&item=7336&utag=
>>http://forums.ecomm.com.br/cgi/dnewsweb.exe?cmd=article&group=forum.carosamigos&item=7428&utag=
http://www.adital.com.br/site/noticia.asp?lang=PT&cod=24972
http://www.adital.com.br/site/noticia.asp?lang=PT&cod=24993http://www.adital.com.br/site/noticia.asp?lang=PT&cod=24973
http://www.adital.com.br/site/noticia.asp?lang=PT&cod=24969
http://www.adital.com.br/site/noticia.asp?lang=PT&cod=24964http://www.adital.com.br/site/noticia.asp?lang=PT&cod=24963
http://www.adital.com.br/site/noticia.asp?lang=PT&cod=24962
http://www.adital.com.br/site/noticia.asp?lang=PT&cod=22858O que acham?___
>>O Yahoo! está de cara nova. Venha conferir!>>http://br.yahoo.com=
>>Instruções para entrar na lista, sair da lista e usar a lista em>>http://www.mat.puc-rio.br/~nicolau/olimp/obm-l.html>>=
>>-->Fernando A Candeias_O Windows Live Spaces é seu espaço na internet com fotos (500 por mês), blog
e agora com rede social http://spaces.live.com/=Instruções para entrar na lista, sair da lista e usar a lista em
http://www.mat.puc-rio.br/~nicolau/olimp/obm-l.html=
-- Be Free Use LINUX 


Re: [obm-l] Comunicado

2006-09-14 Por tôpico Fabio Henrique
É triste , porém a puniçao foi exemlar como deve ser.On 9/14/06, Artur Costa Steiner <[EMAIL PROTECTED]
> wrote:Este fato eh ainda mais triste e lamentavel por envolver um estudante de
nivel medio, muito provavelmente um adolescente, alguem numa idade em quecostuma haver idealismo. Reflexo, talvez, do que acontece em nosso pais.Esperemos que este triste episodio ao menos sirva para que este estudante
reflita e muda suas atitudes para o futuro.Artur-Mensagem original-De: [EMAIL PROTECTED] [mailto:
[EMAIL PROTECTED]]Emnome de Olimpiada Brasileira de MatematicaEnviada em: quinta-feira, 14 de setembro de 2006 02:47Para: Lista de discussaoAssunto: [obm-l] Comunicado*
COMUNICADO*Rio de Janeiro, 13 de Setembro de 2006À Comunidade OlímpicaOlimpíada Brasileira de Matemática - OBMPrezados alunos, professores e comunidade olímpica:
No dia 3 de Setembro de 2006, a Olimpíada Brasileira de Matemáticarecebeu, via e-mail, numerosas denúncias segundo as quais um estudantede Ensino Médio, participante do Nível 3 na XXVIII Olimpíada Brasileira
de Matemática – OBM, tentou de forma ilícita conseguir respostas paraduas questões da prova da OBM, que foi aplicada em todo o territórionacional dia 2/09/2006. Após uma rigorosa investigação e com base nas
provas apresentadas, ficou demonstrado que o aluno, devidamenteidentificado, teve acesso às duas questões da prova diretamente nocolégio onde estuda e que tentou conseguir as soluções postando oconteúdo para uma comunidade existente em um conhecido site de
relacionamentos na Internet.Diante desta grave quebra de sigilo por parte da instituição, e da clarae total violação a todos os princípios de honra olímpica por parte doaluno, a Comissão Nacional de Olimpíadas de Matemática determinou a
imediata desclassificação do aluno, bem como também a desclassificaçãoda instituição de ensino na XXVIII Olimpíada Brasileira de Matemática de2006. Ficou também determinada a proibição, tanto para o aluno quanto
para a instituição, de participar da competição durante o ano de 2007 emtodos os seus níveis de participação. A medida adotada pela ComissãoNacional de Olimpíadas de Matemática é irrevogável, não cabendo recursos
por parte dos envolvidos.Espera-se que atitudes como as anteriormente mencionadas não voltem aocorrer dentro da comunidade olímpica, e ressaltamos que a OlimpíadaBrasileira de Matemática é uma atividade de livre participação, tanto
para os alunos quanto para os professores, e que tem como finalidadeprincipal estimular o estudo da Matemática entre os jovens, aperfeiçoarprofessores e propiciar uma melhoria do ensino e do aprendizado desta
matéria nas escolas brasileiras.Cordialmente,Comissão Nacional de Olimpíadas de MatemáticaOlimpíada Brasileira de Matemática - OBM=
Instruções para entrar na lista, sair da lista e usar a lista emhttp://www.mat.puc-rio.br/~nicolau/olimp/obm-l.html=
=Instruções para entrar na lista, sair da lista e usar a lista emhttp://www.mat.puc-rio.br/~nicolau/olimp/obm-l.html
=-- Be Free Use LINUX


Re: [obm-l] Fabio Henrique equivocado

2004-09-27 Por tôpico Fabio Henrique
 Desculpem. Foi por puro zelo pela nossa lista. Afinal, já aconteceu antes. 
Foi um comentário do tipo "li, entendi errado e não gostei". 



Em 27 Sep 2004, [EMAIL PROTECTED] escreveu: 


>O objetivo desta lista é disseminar matemática de todas as formas sempre 
com 
>respeito em todos os aspectos. Creio que o O Fábio se equivocou devido a a 
>contecimentos recentes. O que eu mandei são livros disponibilizados pelos 
>seus autores. E como todos nós precisamos ler muito para continuarmos 
>ajudando uns aos outros, essa foi minha intenção e creio que é a intenção 
>dos autores. O própio Professor Nicolau vez ou outra oferece seu livro 
nesta 
>lista e onde está erro? 
>Creio que o comentário foi algo do tipo " não li e não gostei" 
>(^_^) 
> 
>_ 
>MSN Messenger: converse com os seus amigos online. 
>http://messenger.msn.com.br 
> 
>Instruções 
>para entrar na lista, sair da lista e usar a lista em 
>http://www.mat.puc-rio.br/~nicolau/olimp/obm-l.html 
> 
> 
>-- 

_
Quer mais velocidade?
Só com o acesso Aditivado iG, a velocidade que você quer na hora que você precisa.
Clique aqui: http://www.acessoaditivado.ig.com.br



Re: [obm-l] Dentro- da- Lei... Livros Gratuitos

2004-09-26 Por tôpico Fabio Henrique
 Será que você poderia divulgar este tipo de material em particular, entre 
os seus. Ou quem sabe, inaugurar uma página para este tipo de prática. 
Talvez você não tenha se dado conta dos problemas que esta lista pública 
pode ter por causa de procedimentos como este. 
Grato. 
Fabio Henrique 


Em 26 Sep 2004, [EMAIL PROTECTED] escreveu: 


>Não é preciso ser fora-da-lei para se conseguir muitos livros, apostilas e 
>monografias gratuitas de Matemática na internet. Aí vão alguns endereços 
>e... divirtam-se 
> 
>http://www.elprisma.com/apuntes/apuntes.asp?categoriap4 
> 
>http://www.numbertheory.org/ntw/lecture_notes.html 
> 
>http://www.math.miami.edu/~ec/book/ 
> 
>http://www.maths.nott.ac.uk/personal/jec/courses/G13NUM/#link 
> 
>http://f2.org/links/books.html 
> 
>Tem mais outros, vou procurar com mais paciência e qualquer dia coloco na 
>lista 
> 
>(^_^) 
> 
>_ 
>MSN Messenger: converse com os seus amigos online. 
>http://messenger.msn.com.br 
> 
>Instruções 
>para entrar na lista, sair da lista e usar a lista em 
>http://www.mat.puc-rio.br/~nicolau/olimp/obm-l.html 
> 
> 
>-- 

_
Quer mais velocidade?
Só com o acesso Aditivado iG, a velocidade que você quer na hora que você precisa.
Clique aqui: http://www.acessoaditivado.ig.com.br



Re: RES: [obm-l] [PELEJA] Desafio do Google

2004-09-18 Por tôpico Fabio Henrique
 Onde posso obter o mathematica? 


Em 18 Sep 2004, [EMAIL PROTECTED] escreveu: 


>Bom, no meu caso eu reconheci os primeiros números como as casas 
>iniciais de e e depois usei o Mathematica para verificar de onde saiam 
>as outras. 
>Daí, foi só perceber a lógica envolvida e procurar a 720ª casa 
>decimal... 
> 
>Um abraço, 
> 
>Guilherme. 
> 
>-Mensagem original- 
>De: [EMAIL PROTECTED] [mailto:[EMAIL PROTECTED] Em 
>nome de David M. Cardoso 
>Enviada em: sábado, 18 de setembro de 2004 01:00 
>Para: [EMAIL PROTECTED] 
>Assunto: Re: [obm-l] [PELEJA] Desafio do Google 
> 
>Como faz pra achar isso? 
>Apelação/Força bruta computacional? 
>E se for um problema te testar exaustivamente vários números (problema 
>computacional), como fazer pra encontrar a n-ésima casa decimal da 
>constante de euler? 
> 
>[]'s 
>David 
> 
> 
>= 
>Instruções para entrar na lista, sair da lista e usar a lista em 
>http://www.mat.puc-rio.br/~nicolau/olimp/obm-l.html 
> 
>= 
> 
>= 
>Instruções para entrar na lista, sair da lista e usar a lista em 
>http://www.mat.puc-rio.br/~nicolau/olimp/obm-l.html 
>= 
> 
>-- 

_
Quer mais velocidade?
Só com o acesso Aditivado iG, a velocidade que você quer na hora que você precisa.
Clique aqui: http://www.acessoaditivado.ig.com.br



Re: [obm-l] Re: IME

2004-07-23 Por tôpico Fabio Henrique
Se não for incômodo, agradeceria muito. 


Em 23 Jul 2004, [EMAIL PROTECTED] escreveu: 


>Fábio. 
> 
> Eu consegui entrar e baixar o arquivo. 
> Zipado, o tamanho é de cerca de 650 kb. 
> Se você quiser, posso enviar para você. 
> 
> Abraços. 
> Hugo. 
> 
> Fabio Henrique wrote: 
> 
>Wallace, é exatamente este endereço que tenho. No entanto, não consigo 
>entrar na página de jeito nenhum. 
>Veja se consegue e me dê retorno. 
>Obrigado. 
> 
>-- 

_
Quer mais velocidade?
Só com o acesso Aditivado iG, a velocidade que você quer na hora que você precisa.
Clique aqui: http://www.acessoaditivado.ig.com.br



Re: [obm-l] Re: IME

2004-07-23 Por tôpico Fabio Henrique
Wallace, é exatamente este endereço que tenho. No entanto, não consigo 
entrar na página de jeito nenhum. 
Veja se consegue e me dê retorno. 
Obrigado. 


Em 21 Jul 2004, [EMAIL PROTECTED] escreveu: 


>Ola Fabio, 
> 
> A pagina do Prof. Ph.D Sergio Lima Netto é a seguinte: 
>http://www.lps.ufrj.br/~sergioln/ 
> 
>Um abraço! 
> 
>Wallace Alves Martins 
>Laboratorio de Processamento de Sinais/UFRJ 
> 
>Fabio Henrique escreve: 
> 
>> Alguém poderia me escrever o endereço da página do professor Sérgio que 
>> contém as provas do Ime? 
>> Grato. 
>> 
>> 
>_ 
>> Quer mais velocidade? 
>> Só com o acesso Aditivado iG, a velocidade que você quer na hora que você 
>precisa. 
>> Clique aqui: http://www.acessoaditivado.ig.com.br 
>> 
> 
>= 
>Instruções para entrar na lista, sair da lista e usar a lista em 
>http://www.mat.puc-rio.br/~nicolau/olimp/obm-l.html 
>= 
> 
>-- 

_
Quer mais velocidade?
Só com o acesso Aditivado iG, a velocidade que você quer na hora que você precisa.
Clique aqui: http://www.acessoaditivado.ig.com.br



[obm-l] IME

2004-07-21 Por tôpico Fabio Henrique
Alguém poderia me escrever o endereço da página do professor Sérgio que 
contém as provas do Ime? 
Grato. 

_
Quer mais velocidade?
Só com o acesso Aditivado iG, a velocidade que você quer na hora que você precisa.
Clique aqui: http://www.acessoaditivado.ig.com.br



[obm-l] Re: ção do teorema de Fermat

2004-06-11 Por tôpico Fabio Henrique
Pode mandar para mim? 
Grato. 


Em 10 Jun 2004, [EMAIL PROTECTED] escreveu: 

>Como faço para disponibilizar o arquivo para o grupo?? 
>Formato: pdf 
>Tamanho: 864 Kb 
>Abraço. 
> 
>= 
>Instruções para entrar na lista, sair da lista e usar a lista em 
>http://www.mat.puc-rio.br/~nicolau/olimp/obm-l.html 
>= 
> 
>-- 

_
Quer mais velocidade?
Só com o acesso Aditivado iG, a velocidade que você quer na hora que você precisa.
Clique aqui: http://www.acessoaditivado.ig.com.br



Re: [obm-l] Teo. de Wilson

2004-06-06 Por tôpico Fabio Henrique
Vou começar com um exemplo numérico. 

Seja p=11 

(p-1)! = 10.9.8.7.6.5.4.3.2.1 

Observe que 9.5 = 1 (mod 11) ENTENDA O SINAL DE = COMO CONGRUENTE. 
8.7 = 1 (mod 11) 
6.2 = 1 (mod 11) 
3.4 = 1 (mod 11) 
Assim, (p-1)! = 10.1.1.1.1.1 = -1.1.1.1.1.1 = -1 

Para p primo qualquer, sabemos que todos os elementos de {1,2,3,4,...,p-1} 
têm inverso multiplicativo. Além disso, o inverso de 1 e 1 (mod p) e o 
inverso de p-1 é p-1 (mod p). Assim, 2.3.4. ... .p-2 = 1 (mod p) 

(p-1)! = 1.2.3. ... .p-2.p-1 = 1.1.1. ... .1.p-1 (mod p) 
(p-1)! = 1.1.1. ... -1 (mod p) 

Em  6 Jun 2004, [EMAIL PROTECTED] escreveu: 

>Pessoal, como provo o teo. de wilson,ou seja, 
>se p é primo entao (p-1)!+1 é congruente a 0 módulo p 
> 
>Atenciosamente, 
> 
>Engenharia Elétrica - UNESP Ilha Solteira 
>Osvaldo Mello Sponquiado 
>Usuário de GNU/Linux 
> 
>__ 
>Acabe com aquelas janelinhas que pulam na sua tela. 
>AntiPop-up UOL - É grátis! 
>http://antipopup.uol.com.br/ 
> 
>= 
>Instruções para entrar na lista, sair da lista e usar a lista em 
>http://www.mat.puc-rio.br/~nicolau/olimp/obm-l.html 
>= 
> 
>-- 

_
Quer mais velocidade?
Só com o acesso Aditivado iG, a velocidade que você quer na hora que você precisa.
Clique aqui: http://www.acessoaditivado.ig.com.br



Re: [obm-l] iso-8859-1?b?UmU6W29ibS1sXSB SZTogW29ibS1sXSBSZTpbb2JtLWx dIFJlOiBbb2Jt iso-8859-1?b?LWxdIFJlOiBbb2I gbS1sXSBSZTpbb2JtLWxdIFJlOiB bb2IgbS1sXSBS iso-8859-1?b?ZTpbb2JtLWwgXSB Db2zpZ2lvIE5hdmFs

2004-06-04 Por tôpico Fabio Henrique
Se não for incômodo... 
Obrigado. 



Em  3 Jun 2004, [EMAIL PROTECTED] escreveu: 

>Que nd, com maior prazer... ja anotei o mail de vcs... 
>matematica sempre convem! 
>falow 
> 
>[Ao som de Numb - Linkin Park] 
> 
>> Oi Osvaldo se não for exploração , quando mandar 
>para os colegas no final da 
>> semana ,gostaria muito que mandaçe pra mim tb, 
>dedsde ja agradeço.Meu 1/2 é 
>> [EMAIL PROTECTED] 
>> - Original Message - 
>> From: "Osvaldo" <[EMAIL PROTECTED]> 
>> To: "obm-l" 
>> Sent: Wednesday, June 02, 2004 8:52 PM 
>> Subject: [obm-l] Re:[obm-l] Re: [obm-l] Re: [obm-l] 
>Re:[obm-l] Re: [ob m-l] 
>> Re:[obm-l] Colégio Naval 
>> 
>> 
>> > ta legal... mando sim.. so no fim d semana! 
>> > falow 
>> > 
>> > 
>> > 
>> > > Se não for muito incomodo, poderia também mandar 
>para 
>> > este e-mail 
>> > > [EMAIL PROTECTED] 
>> > > 
>> > > Agradeço desde de já. 
>> > > 
>> > > - Original Message - 
>> > > From: João Luís 
>> > > To: [EMAIL PROTECTED] 
>> > > Sent: Wednesday, June 02, 2004 9:50 AM 
>> > > Subject: [obm-l] Re: [obm-l] Re:[obm-l] Re: 
>[obm-l] 
>> > Re:[obm-l] Colégio Naval 
>> > > 
>> > > 
>> > > Ok, muito obrigado. Qualquer coisa, pode ir 
>> > mandando aos poucos, se não for te dar muito 
>trabalho. 
>> > Mesmo porque, aqui não é banda larga não, hehehe.. 
>> > > - Original Message - 
>> > > From: Osvaldo 
>> > > To: obm-l 
>> > > Sent: Tuesday, June 01, 2004 10:29 PM 
>> > > Subject: [obm-l] Re:[obm-l] Re: [obm-l] Re: 
>[obm- 
>> > l] Colégio Naval 
>> > > 
>> > > 
>> > > E ai Joao! 
>> > > Vo tentar mandar ao seu e mail no fim de 
>semana, 
>> > pq eu 
>> > > levo o pc no meu primo... e la é banda larga. 
>> > Falou! 
>> > > 
>> > > 
>> > > 
>> > > > Olá pessoal. 
>> > > > Aproveitando a deixa: como sou professor de 
>> > > Matemática (em Belo Horizonte), interesso-me 
>e 
>> > muito 
>> > > por provas de vestibulares e materail afim. A 
>> > todos 
>> > > vocês que têm essas "coleções", como o 
>Osvaldo, 
>> > ou que 
>> > > possam me fornecer qualquer fonte de onde 
>> > encontrar 
>> > > esse material, eu agradeceria muito que me 
>> > enviasse. 
>> > > Não sei se essa lista é exatamente o espaço 
>para 
>> > isso, 
>> > > então podem mandar em PVT para João Luís, 
>> > > [EMAIL PROTECTED] 
>> > > > Agradeço antecipadamente a todos que 
>> > colaborarem!!! 
>> > > > - Original Message - 
>> > > > From: Osvaldo 
>> > > > To: obm-l 
>> > > > Sent: Sunday, May 30, 2004 4:35 AM 
>> > > > Subject: [obm-l] Re:[obm-l] Colégio Naval 
>> > > > 
>> > > > 
>> > > > Boa Fábio, acessei seu site, me rendeu 
>mais 
>> > algumas 
>> > > > provas para minha coleçao em meu 
>winchester. 
>> > falow 
>> > > ai! 
>> > > > 
>> > > > 
>> > > > 
>> > > > > Disponibilizei 13 anos de provas do 
>colégio 
>> > Naval. 
>> > > > > Quando tiver tempo, coloco mais. 
>> > > > > 
>> > > > > Espero que seja útil. 
>> > > > > 
>> > > > > 
>> > > 
>> > http://construtor.aprendebrasil.com.br/fabio1766469 
>> > > > > 
>> > > > > 
>> > > > 
>> > > > Atenciosamente, 
>> > > > 
>> > > > Engenharia Elétrica - UNESP Ilha Solteira 
>> > > > Osvaldo Mello Sponquiado 
>> > > > Usuário de GNU/Linux 
>> > > > 
>> > > > 
>> > > > 
>> > > > 
>> > > 
>> > 
>___ 
>_ 
>> > > __ 
>> > > > Acabe com aquelas janelinhas que pulam 
>na sua 
>> > tela. 
>> > > > AntiPop-up UOL - É grátis! 
>> > > > http://antipopup.uol.com.br/ 
>> > > > 
>> > > > 
>> > > > 
>> > > > 
>> > > 
>> > 
>=== 
>= 
>> > > = 
>> > > > Instruções para entrar na lista, sair da 
>> > lista e 
>> > > usar a lista em 
>> > > > http://www.mat.puc- 
>rio.br/~nicolau/olimp/obm- 
>> > l.html 
>> > > > 
>> > > 
>> > 
>=== 
>= 
>> > > = 
>> > > > 
>> > > 
>> > > Atenciosamente, 
>> > > 
>> > > Engenharia Elétrica - UNESP Ilha Solteira 
>> > > Osvaldo Mello Sponquiado 
>> > > Usuário de GNU/Linux 
>> > > 
>> > > 
>> > > 
>> > > 
>> > 
>___ 
>_ 
>> > __ 
>> > > Acabe com aquelas janelinhas que pulam na sua 
>> > tela. 
>> > > AntiPop-up UOL - É grátis! 
>> > > http://antipopup.uol.com.br/ 
>> > > 
>> > > 
>> > > 
>> > > 
>> > 
>=== 
>= 
>> > = 
>> > > Instruções para entrar na lista, sair da 
>lista e 
>> > usar a lista em 
>> > > http://www.mat.puc-rio.br/~nicolau/olimp/obm- 
>> > l.html 
>> > > 
>> > 
>=== 
>= 
>> > = 
>> > > 
>> > 
>> > Atenciosamente, 
>> > 
>> > Engenharia Elétrica - UNESP Ilha Solteira 
>> > Osvaldo Mello Sponquiado 
>> > Usuário de GNU/Linux 
>> > 
>> > 
>> > 
>> > 
>___ 
>___ 
>> > Acabe com aquelas janelinhas que pulam na sua tela. 
>> > AntiPop-up UOL - É gr

Re:[obm-l] elipse

2004-06-01 Por tôpico Fabio Henrique
Vocês não receberam a solução sem uso de derivadas? 



Em 1 Jun 2004, [EMAIL PROTECTED] escreveu: 

>É claro que não está certo, até porque as equações encontradas não 
representam retas. 
> 
> == 
> Mensagem  enviada  pelo  CIP  WebMAIL  - Nova Geração - v. 2.1 
> CentroIn Internet Provider           http://www.centroin.com.br 
> Tel: (21) 2542-4849, (21) 2295-3331        Fax: (21) 2295-2978 
> Empresa 100% Brasileira - Desde 1992 prestando servicos online 
> 
> -- Original Message --- 
> From: Jefferson Franca 
> To: [EMAIL PROTECTED] 
> Sent: Mon, 31 May 2004 22:21:51 -0300 (ART) 
> Subject: Re:[obm-l] elipse 
> 
> > Valeu! Uma curiosidade: E sem derivada? Como ficaria? 
> > 
> > Osvaldo <[EMAIL PROTECTED]> wrote: Posso decompor esta eq. ai em duas 
funçoes 
> > f(x)_1 = +sqrt(1-(x/2)^2) 
> > f(x)_2 = -sqrt(1-(x/2)^2) 
> > (x_0,y_o)=(3,2) 
> > Uma saída é utilizar y-y_0=y'.(x-x_0) (y'=d(f(x))/dx) 
> > como reta tangente em (x_0,y_0) 
> > 
> > Da primeira funçao vem que y-2=-x(x-3)/sqrt(1-(x/2)^2) 
> > Da segunda funçao vem que y-2=x(x-3)/sqrt(1-(x/2)^2) 
> > 
> > Bom, não sei se ta certo, se estiver a eq. vai 
> > corresponder a 1-(x/2)^2=x(x-3)/(y-2) 
> > 
> > falow ai 
> > 
> > > Será q alguém poderia me ajudar com a questão: 
> > Determine a equação das tangentes à elipse (x^2)/4 + 
> > (y^2) = 1, que passam pelo ponto P(3,2). 
> > > 
> > > 
> > > 4-x^2 /4 -2x 
> > 
> > - 
> > > 
> > > - 
> > > Yahoo! Mail - Participe da pesquisa global sobre o 
> > Yahoo! Mail. Clique aqui! 
> > 
> > Atenciosamente, 
> > 
> > Engenharia Elétrica - UNESP Ilha Solteira 
> > Osvaldo Mello S! ponquiado 
> > Usuário de GNU/Linux 
> > 
> > 
__ 
> > Acabe com aquelas janelinhas que pulam na sua tela. 
> > AntiPop-up UOL - É grátis! 
> > http://antipopup.uol.com.br/ 
> > 
> > 
= 
> > Instruções para entrar na lista, sair da lista e usar a lista em 
> > http://www.mat.puc-rio.br/~nicolau/olimp/obm-l.html 
> > 
= 
> > 
> > 
> Yahoo! Mail - Participe da pesquisa global sobre o Yahoo! Mail. Clique 
aqui! 
> --- End of Original Message --- 
> 
>-- 

_
Quer mais velocidade?
Só com o acesso Aditivado iG, a velocidade que você quer na hora que você precisa.
Clique aqui: http://www.acessoaditivado.ig.com.br



Re: [obm-l] Problema_de_combinatória

2004-05-31 Por tôpico Fabio Henrique
Na verdade você quer saber quantos números são divisíveis por 6 entre 100 e 
999. Nesta faixa, o 1º múltiplo de 6 é 102=6x17 e o último é 996=6x166. 
Agora conte quantos números você tem de 17 a 166. Resp: 166-17+1=150 
Forte abraço. 
Fabio Henrique. 


Em 30 May 2004, [EMAIL PROTECTED] escreveu: 

>Olá pessoal, é um prazer participar desta 
>lista. 
> 
> Resolvi o problema abaixo dividindo-o em muitos 
>casos. 
> 
> "Quantos números de 3 algarismos distintos são 
>divisíveis por 6?" 
> 
> Peço sugestões para uma resolução mais 
>suscinta. 
> 
> Agradeço 
> 
>-- 

_
Quer mais velocidade?
Só com o acesso Aditivado iG, a velocidade que você quer na hora que você precisa.
Clique aqui: http://www.acessoaditivado.ig.com.br



Re: [obm-l] elipse

2004-05-29 Por tôpico Fabio Henrique
Sua reta Y=Ax+B passa pelo ponto (3,2). Assim, B = 2-3A. 

Resolva o sistema formado pela equação da elipse e pela equação da reta (não 
esqueça que B=2-3A). Você cairá em uma equação do 2º grau. Para que a reta 
seja tangente, delta deve ser nulo. Se delta negativo, a reta é exterior sem 
interseção alguma e, quando delta positivo, a reta atravessa a elipse. 
Eu achei y = [(24+sqrt21).x + (2-3sqrt21)]/37 
e y = [(24-sqrt21).x + (2+3sqrt21)]/37 




Em 29 May 2004, [EMAIL PROTECTED] escreveu: 

>Será q alguém poderia me ajudar com a questão: Determine a equação das 
tangentes à elipse (x^2)/4 + (y^2) = 1, que passam pelo ponto P(3,2). 
> 
>-- 

_
Quer mais velocidade?
Só com o acesso Aditivado iG, a velocidade que você quer na hora que você precisa.
Clique aqui: http://www.acessoaditivado.ig.com.br



Re: [obm-l] Re:_CONSELHOS_PEDAGÓGICOS_[o ff-topic]

2004-05-29 Por tôpico Fabio Henrique
Eric, poderia enviar para mim também? 
Obrigado. 
FH 


Em 29 May 2004, [EMAIL PROTECTED] escreveu: 

>> o ensino lá é precário, os 
>> alunos estão vendo 
>> trigonometria sem saber dividisão!!! Não sabem 
>> tabuada Nada, nada, nada. 
> 
>Tenho um material estilo "metodo Kumon" que 
>talvez pudesse ser usado, caso os alunos (ou a escola) 
>estivessem dispostos a pagarem as xerox. Estou 
>comecando a usar este sistema com um sobrinho meu. 
>Posso te mandar alguma coisa nos formatos .xls ou .pdf 
>(ou .doc se tiver Word) 
>te mandar alguma 
> 
>[ ]'s 
> 
>Eric. 
> 
>> Fiquei extremamente triste com a situação e quero 
>> tentar mudá-la. Não sei 
> > como devo proceder, pois eles não têm base alguma... 
> >Por onde devo começar? Tabuada, soma??? (...) 
> >Estou perdido, completamente perdido... 
> >Obrigado, 
> 
>= 
>Instruções para entrar na lista, sair da lista e usar a lista em 
>http://www.mat.puc-rio.br/~nicolau/olimp/obm-l.html 
>= 
> 
>-- 

_
Quer mais velocidade?
Só com o acesso Aditivado iG, a velocidade que você quer na hora que você precisa.
Clique aqui: http://www.acessoaditivado.ig.com.br



Re: [obm-l] CONSELHOS PEDAGÓGICOS [off-topic]

2004-05-29 Por tôpico Fabio Henrique
Sinceramente, acho que você deveria ensinar aos alunos a tal da divisão. É 
mais importante que trigonô. 


Em 28 May 2004, [EMAIL PROTECTED] escreveu: 

>Olá amigos da lista, 
> 
>estou diante de uma situação-problema e gostaria muito de receber uma 
sugestão. 
> 
>Estou fazendo estágio numa escola onde funciona o EJA, educação para jovens 
e adultos. Posso dizer que o ensino lá é precário, os alunos estão vendo 
trigonometria sem saber dividisão!!! Não sabem tabuada Nada, nada, nada. 
> 
>Fiquei extremamente triste com a situação e quero tentar mudá-la. Não sei 
como devo proceder, pois eles não têm base alguma... 
> 
>Por onde devo começar? Tabuada, soma??? (...) 
> 
>Estou perdido, completamente perdido... 
> 
>Obrigado, 
> 
>[]'s 
> 
>-- 

_
Quer mais velocidade?
Só com o acesso Aditivado iG, a velocidade que você quer na hora que você precisa.
Clique aqui: http://www.acessoaditivado.ig.com.br



Re: [obm-l] iso-8859-1?b?UmU6W29ibS1sXSB SZTogW29ibS1sXSBSZTogW29ibS1 sXSBPUyBO2k1F iso-8859-1?b?Uk9TIERPIEFDQVN PIQ==

2004-05-26 Por tôpico Fabio Henrique
Há uma outra interpretação equivocada: 
temos 6 resultados pares (0,2,4,6,8,10) contra 5 ímpares (1,3,5,7,9) 





Em 26 May 2004, [EMAIL PROTECTED] escreveu: 

>Teve uma questão da OBM, acho que foi do ano passado, 
>na prim. fase em que dois jogadores usavam 1,2,... ou 5 
>dedos, e dava 64 por cento de chance de quem escolheu 
>par vencer. 
>Afinal, se cada um usa de 0 a 10 dedos, quem vence? 
> 
>> On Tue, May 25, 2004 at 11:18:00PM -0300, Fellipe 
>Rossi wrote: 
>> > "PASMEM! O jogo do par ou ímpar é, sem sombra de 
>dúvidas, favorável a C. 
>> > Abraços!" 
>> > 
>> > Por quê? 
>> > 
>> > Rossi 
>> 
>> Talvez outros adivinhem melhor do que eu, mas eu não 
>tenho a menor idéia 
>> de quem o o que seja este "C." que estaria sendo 
>favorecido. 
>> 
>> Meu melhor palpite é que estamos falando daquele 
>raciocínio 
>> (completamente errado) que diz que PAR tem 
>probabilidade maior 
>> do que ÍMPAR de ganhar, pois para PAR há duas 
>possibilidades 
>> (os dois jogadores jogam números pares; os dois 
>jogadores 
>> jogam números ímpares) enquenato para ÍMPAR só há uma 
>possibilidade 
>> (os dois jogadores escolhem números de paridades 
>diferentes). 
>> 
>> []s, N. 
>> 
> 
>= 
>> Instruções para entrar na lista, sair da lista e usar 
>a lista em 
>> http://www.mat.puc-rio.br/~nicolau/olimp/obm-l.html 
>> 
> 
>= 
>> 
> 
>Atenciosamente, 
> 
>Engenharia Elétrica - UNESP Ilha Solteira 
>Osvaldo Mello Sponquiado 
>Usuário de GNU/Linux 
> 
>__ 
>Acabe com aquelas janelinhas que pulam na sua tela. 
>AntiPop-up UOL - É grátis! 
>http://antipopup.uol.com.br/ 
> 
>= 
>Instruções para entrar na lista, sair da lista e usar a lista em 
>http://www.mat.puc-rio.br/~nicolau/olimp/obm-l.html 
>= 
> 
>-- 

_
Quer mais velocidade?
Só com o acesso Aditivado iG, a velocidade que você quer na hora que você precisa.
Clique aqui: http://www.acessoaditivado.ig.com.br



Re: [obm-l] Problema da raiz

2004-05-12 Por tôpico Fabio Henrique
Como assim, 16? Não concordo. Desta forma, a "minha idade" fica raiz quarta 
de 30. 


m 12 May 2004, [EMAIL PROTECTED] escreveu: 

>Junior said: 
>> Problema: 
>> "Minha idade é a raiz quadrada da raíz quadrada de 14 mais minha idade." 
>> 
>> desenvolvendo achei: 
>> 
>> x^4 - x - 14 = 0 
>> 
>> encontrando as raízes por briot-rufini, achei 2. 
>> 
>> Será que minha equaçao esta certa? 
>> [...] 
> 
>Sim, mas o "x" não é o que você espera que ele seja -- o x é a raiz quarta 
>da idade do indivíduo. Logo a idade é, na realidade, x^4 = 16. 
> 
>[]s, 
> 
>-- 
>Fábio "ctg \pi" Dias Moreira 
> 
>= 
>Instruções para entrar na lista, sair da lista e usar a lista em 
>http://www.mat.puc-rio.br/~nicolau/olimp/obm-l.html 
>= 
> 
>-- 

_
Voce quer um iGMail protegido contra vírus e spams? 
Clique aqui: http://www.igmailseguro.ig.com.br



[obm-l] Re: dúvida

2004-05-12 Por tôpico Fabio Henrique
Achei 365. 

(x+y)(x-y)=27 
x+y=27 e x-y=1 => x=14 e y=13 => x^2+y^2=365 
x+y=9 e x-y=3 => x=6 e y=3 
Em 12 May 2004, [EMAIL PROTECTED] escreveu: 

>a diferença entre o quadrado de dois números 
>naturais é 27. uma ´possível soma dos quadrados desses números : 
> 
> a)529 
> b)625 
> c)729 
> d)841 
> 
>-- 

_
Voce quer um iGMail protegido contra vírus e spams? 
Clique aqui: http://www.igmailseguro.ig.com.br



Re: [obm-l] 8a.cone sul

2004-05-09 Por tôpico Fabio Henrique
O recado foi para o Dirichlet. 


Em 9 May 2004, [EMAIL PROTECTED] escreveu: 

>Em 9 May 2004, [EMAIL PROTECTED] escreveu: 
> 
>>Cara, não leva a mal. 
>>Você continua não colaborando com coisa alguma. Ou você faz observações 
que 
>>não acrescentam nada, ou faz este tipo de intervenção mágica. Isto é pouco 
>>instrutivo. O que são as equações de Pell? Por que você não explica para 
>>todos? Eu não sei o que são estas equações e resolvi. Acho que você é o 
>>maior enrolão. Pare de embromar e faça algo de construtivo como o Buffara, 
>>Steiner, Morgado, Nicolau, Santa Rita, Cyberhelp, etc. 
>>Tenho pena de quem vir a ser seu aluno. 
>>Passe bem. 
> 
>>Não é nada disso, sou muito esforçado e estou tentando aprender, se voçê 
>não pode me ajudar não me desanime.Pena eu tenho de voçê por não me 
conhecer 
>e já estabelecer um pré-conceito desses. 
> 
>>Em 8 May 2004, [EMAIL PROTECTED] escreveu: 
>> 
>>>a=15*t^2-15t-995 
>>>b=15*t^2-20t-992 
>>>c=15*t^2-18t-993 
>>> 
>>> --- Johann Peter Gustav Lejeune Dirichlet 
>>> escreveu: > E 
>>>so usar umas equaçoes de Pell e o problema 
 sai. 
 A verdadeira treta ai e achar Todas e somente 
 TODAS as soluçoes. Encara essa! 
 
 --- [EMAIL PROTECTED] escreveu: > Essa é 
 boa.Demonstrar que existem infinitos 
 > ternos (a,b,c),com a,b,c números 
 > naturais,que satisfazem a 
 > relação:2a^2+3b^2-5c^2=1997.Abraços vieira. 
 > 
 > 
 
>>>_ 
 > Voce quer um iGMail protegido contra vírus e 
 > spams? 
 > Clique aqui: 
 http://www.igmailseguro.ig.com.br 
 > Ofertas imperdíveis! Link: 
 > http://www.americanas.com.br/ig/ 
 > 
 > 
 
>>>= 
 > Instruções para entrar na lista, sair da 
 lista 
 > e usar a lista em 
 > 
 
>>>http://www.mat.puc-rio.br/~nicolau/olimp/obm-l.html 
 > 
 
>>>= 
 
 
 = 
 TRANSIRE SVVM PECTVS MVNDOQVE POTIRI 
 
 CONGREGATI EX TOTO ORBE MATHEMATICI OB SCRIPTA 
 INSIGNIA TRIBVERE 
 
 Fields Medal(John Charles Fields) 
 
 N.F.C. (Ne Fronti Crede) 
 
 
 
 
>>>__ 
 
 Yahoo! Messenger - Fale com seus amigos online. 
 Instale agora! 
 http://br.download.yahoo.com/messenger/ 
 
>>>= 
 Instruções para entrar na lista, sair da lista 
 e usar a lista em 
 
>>>http://www.mat.puc-rio.br/~nicolau/olimp/obm-l.html 
 
>>>= 
>>> 
>>>= 
>>>TRANSIRE SVVM PECTVS MVNDOQVE POTIRI 
>>> 
>>>CONGREGATI EX TOTO ORBE MATHEMATICI OB SCRIPTA INSIGNIA TRIBVERE 
>>> 
>>>Fields Medal(John Charles Fields) 
>>> 
>>>N.F.C. (Ne Fronti Crede) 
>>> 
>>>__ 
>>> 
>>>Yahoo! Messenger - Fale com seus amigos online. Instale agora! 
>>>http://br.download.yahoo.com/messenger/ 
>>>= 
>>>Instruções para entrar na lista, sair da lista e usar a lista em 
>>>http://www.mat.puc-rio.br/~nicolau/olimp/obm-l.html 
>>>= 
>>> 
>>>-- 
>> 
>>_ 
>>Voce quer um iGMail protegido contra vírus e spams? 
>>Clique aqui: http://www.igmailseguro.ig.com.br 
>> 
>>-- 
> 
>_ 
>Voce quer um iGMail protegido contra vírus e spams? 
>Clique aqui: http://www.igmailseguro.ig.com.br 
>Ofertas imperdíveis! Link: http://www.americanas.com.br/ig/ 
> 
>= 
>Instruções para entrar na lista, sair da lista e usar a lista em 
>http://www.mat.puc-rio.br/~nicolau/olimp/obm-l.html 
>= 
> 
>-- 

_
Voce quer um iGMail protegido contra vírus e spams? 
Clique aqui: http://www.igmailseguro.ig.com.br



Re: [obm-l] 8a.cone sul

2004-05-09 Por tôpico Fabio Henrique
Cara, não leva a mal. 
Você continua não colaborando com coisa alguma. Ou você faz observações que 
não acrescentam nada, ou faz este tipo de intervenção mágica. Isto é pouco 
instrutivo. O que são as equações de Pell? Por que você não explica para 
todos? Eu não sei o que são estas equações e resolvi. Acho que você é o 
maior enrolão. Pare de embromar e faça algo de construtivo como o Buffara, 
Steiner, Morgado, Nicolau, Santa Rita, Cyberhelp, etc. 
Tenho pena de quem vir a ser seu aluno. 
Passe bem. 



Em 8 May 2004, [EMAIL PROTECTED] escreveu: 

>a=15*t^2-15t-995 
>b=15*t^2-20t-992 
>c=15*t^2-18t-993 
> 
> --- Johann Peter Gustav Lejeune Dirichlet 
> escreveu: > E 
>so usar umas equaçoes de Pell e o problema 
>> sai. 
>> A verdadeira treta ai e achar Todas e somente 
>> TODAS as soluçoes. Encara essa! 
>> 
>> --- [EMAIL PROTECTED] escreveu: > Essa é 
>> boa.Demonstrar que existem infinitos 
>> > ternos (a,b,c),com a,b,c números 
>> > naturais,que satisfazem a 
>> > relação:2a^2+3b^2-5c^2=1997.Abraços vieira. 
>> > 
>> > 
>> 
>_ 
>> > Voce quer um iGMail protegido contra vírus e 
>> > spams? 
>> > Clique aqui: 
>> http://www.igmailseguro.ig.com.br 
>> > Ofertas imperdíveis! Link: 
>> > http://www.americanas.com.br/ig/ 
>> > 
>> > 
>> 
>= 
>> > Instruções para entrar na lista, sair da 
>> lista 
>> > e usar a lista em 
>> > 
>> 
>http://www.mat.puc-rio.br/~nicolau/olimp/obm-l.html 
>> > 
>> 
>= 
>> 
>> 
>> = 
>> TRANSIRE SVVM PECTVS MVNDOQVE POTIRI 
>> 
>> CONGREGATI EX TOTO ORBE MATHEMATICI OB SCRIPTA 
>> INSIGNIA TRIBVERE 
>> 
>> Fields Medal(John Charles Fields) 
>> 
>> N.F.C. (Ne Fronti Crede) 
>> 
>> 
>> 
>> 
>__ 
>> 
>> Yahoo! Messenger - Fale com seus amigos online. 
>> Instale agora! 
>> http://br.download.yahoo.com/messenger/ 
>> 
>= 
>> Instruções para entrar na lista, sair da lista 
>> e usar a lista em 
>> 
>http://www.mat.puc-rio.br/~nicolau/olimp/obm-l.html 
>> 
>= 
> 
>= 
>TRANSIRE SVVM PECTVS MVNDOQVE POTIRI 
> 
>CONGREGATI EX TOTO ORBE MATHEMATICI OB SCRIPTA INSIGNIA TRIBVERE 
> 
>Fields Medal(John Charles Fields) 
> 
>N.F.C. (Ne Fronti Crede) 
> 
>__ 
> 
>Yahoo! Messenger - Fale com seus amigos online. Instale agora! 
>http://br.download.yahoo.com/messenger/ 
>= 
>Instruções para entrar na lista, sair da lista e usar a lista em 
>http://www.mat.puc-rio.br/~nicolau/olimp/obm-l.html 
>= 
> 
>-- 

_
Voce quer um iGMail protegido contra vírus e spams? 
Clique aqui: http://www.igmailseguro.ig.com.br



[obm-l] Cadê?

2004-05-09 Por tôpico Fabio Henrique
Por onde andará Putinha da Silva? 

_
Voce quer um iGMail protegido contra vírus e spams? 
Clique aqui: http://www.igmailseguro.ig.com.br



Re: [obm-l] COLEGIO NAVAL

2004-05-09 Por tôpico Fabio Henrique
Continuando... 

Há um problema que pede o lado em função do raio R e a resposta é 
L.sqrt(2)/2. 

Em 8 May 2004, [EMAIL PROTECTED] escreveu: 

>leandro-epcar said: 
>> COLÉGIO NAVAL (1987) 
>> 
>> A equação do segundo grau X^2-2X+M=0,m<0,tem raízes X' 
>> e X" ,se X'^(N-2)+X"^(N-2)=A e X'^(N-1)+X"^(N-1) 
>> =B,então X'+X" é: 
>> (A) 2A+MB 
>> (B) 2B-MA 
>> (C) MA+2B 
>> (D) MA-2B 
>> (E) M(A-2B) 
>> [...] 
> 
>Eu suponho que você quer X'^N + X"^N? Caso contrário, é fácil ver que não 
>há solução. Neste caso, se X e Y são as raízes (para simplificar a 
>notação), temos que (x+y)(x^(n-1)+y^(n-1)) = x^n + y^n + 
>xy(x^(n-2)+y^(n-2)), mas a gente sabe quanto valem x+y e xy. 
> 
>[]s, 
> 
>-- 
>Fábio "ctg \pi" Dias Moreira 
> 
>= 
>Instruções para entrar na lista, sair da lista e usar a lista em 
>http://www.mat.puc-rio.br/~nicolau/olimp/obm-l.html 
>= 
> 
>-- 

_
Voce quer um iGMail protegido contra vírus e spams? 
Clique aqui: http://www.igmailseguro.ig.com.br



Re: [obm-l] COLEGIO NAVAL

2004-05-09 Por tôpico Fabio Henrique
Sinceramente, não consigo entender os enunciados. 

Na questão abaixo, quem é N? É um natural qualquer? 

1/M^3 + 1/N^2? 


Em 8 May 2004, [EMAIL PROTECTED] escreveu: 

>leandro-epcar said: 
>> COLÉGIO NAVAL (1987) 
>> 
>> A equação do segundo grau X^2-2X+M=0,m<0,tem raízes X' 
>> e X" ,se X'^(N-2)+X"^(N-2)=A e X'^(N-1)+X"^(N-1) 
>> =B,então X'+X" é: 
>> (A) 2A+MB 
>> (B) 2B-MA 
>> (C) MA+2B 
>> (D) MA-2B 
>> (E) M(A-2B) 
>> [...] 
> 
>Eu suponho que você quer X'^N + X"^N? Caso contrário, é fácil ver que não 
>há solução. Neste caso, se X e Y são as raízes (para simplificar a 
>notação), temos que (x+y)(x^(n-1)+y^(n-1)) = x^n + y^n + 
>xy(x^(n-2)+y^(n-2)), mas a gente sabe quanto valem x+y e xy. 
> 
>[]s, 
> 
>-- 
>Fábio "ctg \pi" Dias Moreira 
> 
>= 
>Instruções para entrar na lista, sair da lista e usar a lista em 
>http://www.mat.puc-rio.br/~nicolau/olimp/obm-l.html 
>= 
> 
>-- 

_
Voce quer um iGMail protegido contra vírus e spams? 
Clique aqui: http://www.igmailseguro.ig.com.br



Re: [obm-l] cococolegio navalvalval

2004-05-09 Por tôpico Fabio Henrique
Há um erro no final da solução ou estou equivocado? 
Se M^3=1000-3(10)-N^3, então M^3+N^3=970. 

Em  8 May 2004, [EMAIL PROTECTED] escreveu: 

>Eu sei que estou sendo incoveniente mas essas 
>quetões que eu venho mandando são duvidas de acumuladas 
>de 2 anos,ficarei muito grato se voçês me ajudarem. 
> 
> COLÉGIO NAVAL (1989) 
> 
> Sendo M e N as raízes da equação X^2-10X+1=0 , o 
>valor da expressão 1/M^3 + 1/N^2 é : 
> 
> (A) 970 
> (B) 950 
> (C) 920 
> (D) 900 
> (E) 870 
>=== 
> OBSERVAÇÃO: 
> 
> Fazendo as raízes da equação ,encontraremos {5-2[6^ 
>(1/2)]} e {5+2[6^(1/2)]} 
> 
> fazendo de acordo com o enunciado teremos: 
> 
> 1 + 1 
>  = 485+198(6^(1/2)) 
>{5-2[6^(1/2)]}^3} {5+2[6^(1/2)]}^2} 
> 
> Oque se aproxíma de 970,mas a resposta não é 
>aproximada ,é exata e se voçê inverter os valores de M 
>e N, o resultado será difernte. 
>== 
> Fiz uma anologia más não ajudou muito 
> 
> (M+N)^3=M^3+N^3+3(M^2)N+3(N^2)M 
> 
> o que se conclui que : 
> 
> M^3=(M+N)^3-3MN(M+N)-N^3 
> 
> o que se conclui que: 
> 
> M^3=1000-3(10)-N^3 
> M^3-n^3=970 
> O QUE SE TORNOU ESTRANHO É CONCLUIR QUE : 
> 
> 1/(m^3)+1/(n^2) = m^3-n^3 
>=== 
> 
>__ 
>Acabe com aquelas janelinhas que pulam na sua tela. 
>AntiPop-up UOL - É grátis! 
>http://antipopup.uol.com.br/ 
> 
>= 
>Instruções para entrar na lista, sair da lista e usar a lista em 
>http://www.mat.puc-rio.br/~nicolau/olimp/obm-l.html 
>= 
> 
>-- 

_
Voce quer um iGMail protegido contra vírus e spams? 
Clique aqui: http://www.igmailseguro.ig.com.br



Re: [obm-l] Re: [obm-l] Princípio_de_Dirich let

2004-05-09 Por tôpico Fabio Henrique
Vocês não acham que o enunciado era para ser: "utilizando-se duas cores"? 


Em 09 May 2004, [EMAIL PROTECTED] escreveu: 

>Fabiano Sant'Ana wrote: 
> 
>> como um simples ponto poderá possuir Duas cores? 
> 
> Desenha um círculo no papel, aí pra cada ponto 
>do círculo você pinta de azul por cima da folha, e de 
>vermelho na parte de baixo. Pronto, agora um simples 
>ponto tem duas cores! 
> 
> Eu aqui estou trabalhando profissionalmente 
>com coisa muito mais bizarra, que é ponto orientado 
>(na verdade, vetor 2D degenerado cuja magnitude foi 
>pra zero, mas preservou a direção e sentido). 
> 
> 
>Ricardo Bittencourt http://www.mundobizarro.tk 
>[EMAIL PROTECTED] "tenki ga ii kara sanpo shimashou" 
>-- União contra o forward - crie suas proprias piadas -- 
>= 
>Instruções para entrar na lista, sair da lista e usar a lista em 
>http://www.mat.puc-rio.br/~nicolau/olimp/obm-l.html 
>= 
> 
>-- 

_
Voce quer um iGMail protegido contra vírus e spams? 
Clique aqui: http://www.igmailseguro.ig.com.br



Re: [obm-l] OT: antispam do UOL

2004-05-03 Por tôpico Fabio Henrique
Já que você tocou no assunto, não faz mesmo sentido entrar em uma lista de 
discussões com anti-spam ativado. 



Em 3 May 2004, [EMAIL PROTECTED] escreveu: 

>favor desligarem seus antispams do UOL pois a cada mensagem que eu mando 
>volta alguma coisa por causa dessa porcaria de antispam nada inteligente do 
>UOL. 
> 
>[ ]'s 
> 
>= 
>Instruções para entrar na lista, sair da lista e usar a lista em 
>http://www.mat.puc-rio.br/~nicolau/olimp/obm-l.html 
>= 
> 
>-- 

_
Voce quer um iGMail protegido contra vírus e spams? 
Clique aqui: http://www.igmailseguro.ig.com.br



Re: [obm-l] Integral

2004-05-02 Por tôpico Fabio Henrique
Nesta mensagem vejo a integral de x.senx. No entanto, em outra mensagem, o 
próprio proponente cita integral de x/senx. Estarei enganado? 


Em 2 May 2004, [EMAIL PROTECTED] escreveu: 

>Faça u = x, dv = senx dx e integração por partes. 
> 
> Obs: Os membros da lista que usam conexão discada amam suas mensagens com 
essas caudas que não dizem respeito a ela. 
> 
> == 
> Mensagem  enviada  pelo  CIP  WebMAIL  - Nova Geração - v. 2.1 
> CentroIn Internet Provider           http://www.centroin.com.br 
> Tel: (21) 2542-4849, (21) 2295-3331        Fax: (21) 2295-2978 
> Empresa 100% Brasileira - Desde 1992 prestando servicos online 
> 
> -- Original Message --- 
> From: Alan Pellejero 
> To: [EMAIL PROTECTED] 
> Sent: Sun, 2 May 2004 17:04:43 -0300 (ART) 
> Subject: [obm-l] Integral 
> 
> > Pessoal, até agora não sei direito o que é off-topic, mas já que não 
consigo resolver essa aqui, gostaria de saber se alguém tem alguma sugestão 
para: 
> >   
> > / 
> > | x(senx)dx 
> > / 
> >   
> > Obrigado 
> > Alan Pellejero 
> > 
> > [EMAIL PROTECTED] wrote: Seja P(x_0, y_0) um ponto qualquer e r:ax+by+c=0 
a equacao da circunferencia, temos que a distancia entre elas eh dada por: 
> > d_p;r = |a*x_0 + b*y_0 + c| / (sqrt(a^2 + b^2)) 
> > 
> > Como sabemos que a origem eh o ponto P(0,0) e r eh 4x + 3y –5 = 0 
> > 
> > d_p;r = |a*x_0 + b*y_0 + c| / (sqrt(a^2 + b^2)) 
> > d_p;r = |4*0 + 3*0 - 5| / (sqrt(4^2 + 3^2)) 
> > d_p;r = 5 / 5 = 1 
> > 
> > Em uma mensagem de 2/5/2004 13:30:14 Hora padrão leste da Am. Sul, 
[EMAIL PROTECTED] escreveu: 
> > 
> > 
> > 
> > Pessoal, como calcular origem de reta em geo. analitica ? 
> > 
> > Ex: Calcule a distância da origem à reta   r: 4x + 3y –5 = 0 
> > 
> > 
> > 
> > 
> > 
> > 
> Yahoo! Messenger - Fale com seus amigos online. Instale agora! 
> --- End of Original Message --- 
> 
>-- 

_
Voce quer um iGMail protegido contra vírus e spams? 
Clique aqui: http://www.igmailseguro.ig.com.br



Re: [obm-l] Conjectura de Goldbach

2004-04-29 Por tôpico Fabio Henrique
Apoiado! 



Em 29 Apr 2004, [EMAIL PROTECTED] escreveu: 

>Olá, amigos. 
> 
>Entrei nessa lista há poucos dias, atraído pela possibilidade 
>de ver boa matemática em ação, e eventualmente esclarecer 
>dúvidas em análise funcional. 
> 
>Porém, depois de mensagens como a que o amigo 234 
>respondeu, fico em dúvida quanto à credibilidade dessa lista. 
> 
>Temos um cara (o que usa MUITO injustamente a alcunha de 
>Dirichlet) que só responde abobrinha e nunca resolve um 
>exercício, um outro cara que acha que consegue conversar 
>a respeito de relatividade como se estivesse num boteco com 
>amigos, outro que manda oitenta mensagens por dia a respeito 
>de integrais, e figuras menos marcantes, mas ainda assim risíveis. 
> 
>Em poucos dias por aqui, vi alguém enunciando a conjectura de 
>Goldbach como se este fosse oriundo de um sanatório, um 
>indivíduo querendo ficar craque em integrais mas não conseguindo 
>resolver o problema da melancia, e por aí vai. 
> 
>Sinceramente, acho que vocês podem fazer melhor que isso se 
>estudarem bastante e se dignarem a manter a boca fechada. 
>Um abraço cordial a todos. 
> 
>Até a próxima, 
> 
>-- Gabriel 
> 
>- Original Message - 
>From: <[EMAIL PROTECTED]> 
>To: 
>Sent: Wednesday, April 28, 2004 11:41 PM 
>Subject: Re: [obm-l] Conjectura de Goldbach 
> 
>> Nossa, usou éter??? 
>> 
>> Um número ímpar: 2k + 1 
>> Outro número ímpar: 2n + 1 
>> 
>> (2k + 1) + (2n + 1) = 2k + 2n + 2 
>> 
>> = 2.(k + n + 1) = múltiplo de 2 = PAR 
>> 
>> 234 
>> 
>> - Original Message - 
>> From: "Everton A. Ramos (www.bs2.com.br)" 
>> To: 
>> Sent: Wednesday, April 28, 2004 10:57 PM 
>> Subject: [obm-l] Conjectura de Goldbach 
>> 
>> 
>> > Boa noite... 
>> > 
>> > "Todo número par é a soma de dois números ímpares" 
>> > 
>> > ??? 
>> > 
>> > Sedo X um número par... (X - 1) será ímpar... então (X - 1) + 1 = X 
>> > 
>> > 1 é ímpar, então... ??? 
>> > 
>> > Porque isso é tão desafiante? 
> 
>= 
>Instruções para entrar na lista, sair da lista e usar a lista em 
>http://www.mat.puc-rio.br/~nicolau/olimp/obm-l.html 
>= 
> 
>-- 

_
Voce quer um iGMail protegido contra vírus e spams? 
Clique aqui: http://www.igmailseguro.ig.com.br



Re: [obm-l] + 1 da sexta série...

2004-04-28 Por tôpico Fabio Henrique
Seja N o número de irmãos e M o número de irmãs. 
Cada filho tem N-1 irmãos e M irmãs ==> N-1 = M 
Cada filha tem N irmãos e M-1 irmãs ==> N = 2(M-1) 
Continue... 


Em 28 Apr 2004, [EMAIL PROTECTED] escreveu: 

>Eu resolvi por um "método" meio louco, gostaria de saber se há resposta 
algébrica para essa coisa aqui: 
> 
>Um casal tem filhos e filhas. Cada filho tem o número de irmãos igual ao 
número de irmãs. Cada filha tem o número de irmãos igual ao dobro do número 
de irmãs. Qual o total de filhos e filhas do casal? 
> 
>Obs: Se alguém me ajudar a resolver esse aqui e o da melancia, dai vou 
poder entrar logo na geometria hiperbólica! 
> 
>Muito obrigado 
> 
>-- 

_
Voce quer um iGMail protegido contra vírus e spams? 
Clique aqui: http://www.igmailseguro.ig.com.br



Re: [obm-l] dúvidazinha!!!!!

2004-04-22 Por tôpico Fabio Henrique
Não deveria me meter nesta pendenga mas já estou a fazê-lo. 
Acho que a maioria das pessoas que andam participando da lista estão se 
acostumando a simplesmente perguntar antes de esgotar as suas próprias 
possibilidades. Este processo me parece totalmente estéril. 



Em 21 Apr 2004, [EMAIL PROTECTED] escreveu: 

>quantos multiplos de 10, 5, 7 existem entre 1 e 
>1000 ? 
> 
>-- 

_
Voce quer um iGMail protegido contra vírus e spams? 
Clique aqui: http://www.igmailseguro.ig.com.br



Re: [obm-l] dúvida

2004-04-03 Por tôpico Fabio Henrique
Observe que M^2 vale, no mínimo, zero. 
E M^2 + N vale, no mínimo, N (quando M=0) 
Pensemos então em f(x)=M^2+N. 
O menor valor que esta parábola assume é 3. Logo, N=3 (e isto acontece 
quando M=0. 
Uma função do segundo grau pode ser pensada sempre como 
f(x)= A.(x-p)^2 + q, onde q é o valor mínimo da função e p, o valor de x que 
proporciona este mínimo. A é aquele mesmo da forma A^2+Bx+C. 

Assim, o seu problema fica f(x)=A.(x-0)^2+3=A.x^2+3 
Observe que y=13 quando x=10. Substituíndo, encontrará A=1/10. 
Esta forma de escrever a função do 2º grau é chamada forma canônica. 
Abraços. 
Fabio Henrique 









um determinado fio é constituído de um material que, quando preso a dois 
>> pontos distantes um do outro de 20m e ambos a 13m do solo, toma a forma 
de 
>> uma parábola, estando o ponto mais baixo do fio a 3m do solo. Assinale a 
>> alternativa que corresponde à parábola no sistema de coordenadas 
>> cartesianas XOY, onde o eixo OY contém o ponto mais baixo do fio e o eixo 
>> OX está sobre o solo. 
>> [...] 
> 
>Inicialmente, note que fios suspensos *não* formam parábolas, mas sim 
>catenárias, que são coisas parecidas com o gráfico de (e^x + e^-x)/2. 
> 
>Pedantismos físicos à parte, note que se os pontos de apoio são os pontos 
>(10, 
>13) e (-10, 13), então o ponto mais baixo é o ponto (0, 3). Associe à 
>parábola uma função f(x) tal que todos os pontos da parábola são da forma 
>(x, 
>f(x)). Portanto, f(10) = f(-10) = 13 e f(0) = 3. 
> 
>Obviamente, f é quadrática. Considere g(x) = f(x) - 13. Então 10 e -10 são 
>dois zeros de g, logo g(x) = a*(x-10)*(x+10), onde a é um real. Como g(0) = 
>- -10, a*(-10)*10 = -10 <=> a = 1/10. Logo g(x) = (x-10)*(x+10)/10, logo 
>f(x) = 
>(x-10)*(x+10)/10 + 13. Como as alternativas da questão não chegaram aqui, 
>essa é a melhor resposta que eu posso dar. 
> 
>[]s, 
> 
>- -- 
>Fábio Dias Moreira 
>http://dias.moreira.nom.br/ 
>-BEGIN PGP SIGNATURE- 
>Version: GnuPG v1.2.3 (GNU/Linux) 
> 
>iD8DBQFAb1YbalOQFrvzGQoRAmytAKDAnMNnuzn97j+I85/F8k+fOiM2xACfbeon 
>dRZpXPhtXJ8ltTc/tINuXZQ= 
>=bpms 
>-END PGP SIGNATURE- 
> 
>= 
>Instruções para entrar na lista, sair da lista e usar a lista em 
>http://www.mat.puc-rio.br/~nicolau/olimp/obm-l.html 
>= 
> 
>-- 

_
Voce quer um iGMail protegido contra vírus e spams? 
Clique aqui: http://www.igmailseguro.ig.com.br



[obm-l] Re: Equaçoes irracionaiis

2004-03-27 Por tôpico Fabio Henrique
Faltou dizer porque: 

sqrt vem de square root. 



Em 27 Mar 2004, [EMAIL PROTECTED] escreveu: 

>Please, qual nomenclatura eu devo utilizar para raíz quadrada e divisão? 
>Em quanto nao sei, vou apenas escreve-la. 
>Me ajudem a resolver esta questao: 
>(Raíz Q x^2+9) - {15 / (Raíz Q x^2+9 ) }= 2 
> 
>_ 
>MSN Messenger: converse com os seus amigos online. 
>http://messenger.msn.com.br 
> 
>Instruções 
>para entrar na lista, sair da lista e usar a lista em 
>http://www.mat.puc-rio.br/~nicolau/olimp/obm-l.html 
> 
> 
>-- 

_
Voce quer um iGMail protegido contra vírus e spams? 
Clique aqui: http://www.igmailseguro.ig.com.br



[obm-l] Re: Equaçoes irracionaiis

2004-03-27 Por tôpico Fabio Henrique
sqrt(x^2+9) 
Eu chegaria a escrever sqrt((x^2)+9) 



Em 27 Mar 2004, [EMAIL PROTECTED] escreveu: 

>Please, qual nomenclatura eu devo utilizar para raíz quadrada e divisão? 
>Em quanto nao sei, vou apenas escreve-la. 
>Me ajudem a resolver esta questao: 
>(Raíz Q x^2+9) - {15 / (Raíz Q x^2+9 ) }= 2 
> 
>_ 
>MSN Messenger: converse com os seus amigos online. 
>http://messenger.msn.com.br 
> 
>Instruções 
>para entrar na lista, sair da lista e usar a lista em 
>http://www.mat.puc-rio.br/~nicolau/olimp/obm-l.html 
> 
> 
>-- 

_
Voce quer um iGMail protegido contra vírus e spams? 
Clique aqui: http://www.igmailseguro.ig.com.br



Re: [obm-l] Pequeno erro (um desafio em calculadoras)

2004-03-20 Por tôpico Fabio Henrique
Rafael. 
Em primeiro lugar, não concordo que o Niski tenha errado o problema por 
causa da aproximação. Ele sabia que 
i >= 4log[2](10) e 
12 < 4log[2](10)< 16 
Isto NÃO QUER DIZER que i >= 16. O i pode estar entre 4log[2](10)e 16. 
Em segundo lugar, pode ser enfadonho calcular a raiz cúbica de um número em 
uma calculadora com 4 operações e raiz quadrada, se tivermos que recorrer a 
isso todas as vezes que este cálculo aparecer. No entanto, acho um desafio 
interessantíssimo e muito instrutivo no que diz respeito aos métodos 
iterativos de resoluções de equações. Pode ser um exemplo interessante para 
se começar uma aula sobre Método de Newton. 
Saudações matemáticas. 
Fabio Henrique 

Em 19 Mar 2004, [EMAIL PROTECTED] escreveu: 

>Bem, o fato é que eu não sugeri em *nenhum* momento que se usasse uma 
>calculadora. A minha idéia era simplesmente mostrar que a aproximação feita 
>do log(10,2) estava gerando o erro. Há pouco li uma mensagem do Ricardo em 
>que ele expôs habilmente como a aproximação estava errada, algo que eu não 
>havia feito. 
> 
>Sobre o problema que você propõe, bem, ele é muito enfadonho. Antes das 
>calculadoras, foram criados algoritmos para a extração de raízes quadradas 
e 
>cúbicas. Eu cheguei a aprender ambos, embora isso só fizesse sentido se 
>ensinado há algumas décadas, e não sou tão jurássico assim. O algoritmo 
para 
>a extração de raízes quadradas é até útil, se você quiser uma aproximação 
de 
>até 2 casas decimais. Para mais do que isso, você já precisará de uma 
>calculadora para realizar as multiplicações gigantescas, subtrações 
>intermediárias etc. Suponho que você tenha se referido ao das raízes 
>cúbicas, mas este é ainda pior nas operações, mesmo que, obviamente, deva 
>ter sido válido mito tempo atrás, embora eu não ache propriamente algo 
>prático ou divertido... 
> 
>- Original Message - 
>From: "Johann Peter Gustav Lejeune Dirichlet" 
> 
>To: 
>Sent: Friday, March 19, 2004 6:43 PM 
>Subject: Re: [obm-l] Pequeno erro (um desafio em calculadoras) 
> 
>A ideia, Rafael, e que nao e muito logico (nao a 
>mentes humanas...) usar a calculadora em apenas 
>uma conta do problema, quando o problema inteiro 
>pode ser feito com uma boa calculadora. 
>Para rechear a mensagem proponho o problema: 
> 
>com uma calculadora que tenha as operaçoes 
>basicas determine a raiz cubica de um numero dado 
>(e claro que nao exijo exatidao de 100 por cento 
>mas de pelo menos todas as cifras da calculadora, 
>o que da 8 ou 10 digitos).Pode ser um metodo 
>iterativo (ter uma aproximaçao e melhora-la, 
>usando a mesma receita). 
>Acho isso facil mas e so para diversao mesmo.. 
> 
>= 
>Instruções para entrar na lista, sair da lista e usar a lista em 
>http://www.mat.puc-rio.br/~nicolau/olimp/obm-l.html 
>= 
> 
>-- 

_
Voce quer um iGMail protegido contra vírus e spams? 
Clique aqui: http://www.igmailseguro.ig.com.br



Re: [obm-l] livros

2004-03-20 Por tôpico Fabio Henrique
Não é verdade. Eu VI E FOLHEEI um esta semana na Galileu do L. Machado. 


Em 18 Mar 2004, [EMAIL PROTECTED] escreveu: 

>Olá Antônio, 
> 
> Por acaso a livraria é: 
> Livraria Academia do Saber 
> Av. Passos, 25 
>Centro 
>Tel: (21) 2242-4826 
>Livros novos e usados 
> 
> http://www.aosaber.com.br/ 
> 
> Infelizmente, esse site está desativado. 
> Espero estar errado... se você tiver o endereço eletrônico, eu agradeço. 
> []´s Nelson 
> 
> p.s.: Realmente, no site da livraria galileu consta os livros, mas eles 
acabaram de me avisar que eles estão na verdade esgotados. 
> 
> Antonio Neto wrote: 
> 
> Por acaso, soube no sábado passado. Se voce estiver no Rio de Janeiro, 
encontrarah uma livraria na Av. Passos, 23 ou 25, mas estah de frente para a 
rua. Encontrei 5 ou 6 exemplares de cada, em bom estado. Sem querer imitar 
nenhum participante da lista, depois eu mando o endereco eletronico, eh que 
estou no trabalho. Abracos, olavo. 
> 
> >From: Nelson 
> >Reply-To: [EMAIL PROTECTED] 
> >To: [EMAIL PROTECTED] 
> >Subject: [obm-l] livros 
> >Date: Wed, 17 Mar 2004 11:36:14 -0300 (ART) 
> > 
> >Olá a todos, 
> > 
> >Alguém poderia me dizer como encontro os livros: 
> >GEOMETRIA I e II, e ALGEBRA I (morgado, a.c., et alii) 
> > 
> >E alguém sabe como entrar em contato com a editora Francisco Alves? 
> > 
> >Desde já, agradeço. 
> >Nelson 
> 
>-- 

_
Voce quer um iGMail protegido contra vírus e spams? 
Clique aqui: http://www.igmailseguro.ig.com.br



Re: [obm-l] livros

2004-03-17 Por tôpico Fabio Henrique
Encontrei um ontem na Galileu do Lgo do Machado. 




Em 17 Mar 2004, [EMAIL PROTECTED] escreveu: 

>Olá a todos, 
> 
> Alguém poderia me dizer como encontro os livros: 
> GEOMETRIA I e II, e ALGEBRA I (morgado, a.c., et alii) 
> 
> E alguém sabe como entrar em contato com a editora Francisco Alves? 
> 
> Desde já, agradeço. 
> 
>-- 

_
Voce quer um iGMail protegido contra vírus e spams? 
Clique aqui: http://www.igmailseguro.ig.com.br



[obm-l] Re: dúvida

2004-03-17 Por tôpico Fabio Henrique
L = V-C = 20%deV = V/5 
Assim, C = 4V/5 

L/C = (V/5)/(4V/5) = 1/4 

So que estes 20% sobre o preco de venda chamam-se MARGEM DE LUCRO. O que 
chamamos LUCRO eh o percentual sobre o preco de custo. 



Em 17 Mar 2004, [EMAIL PROTECTED] escreveu: 

>olá amigos estou com uma dúvida; 
> 
> O lucro obtido na venda de certo artigo corresponde 
>a 20% de seu preço de venda. A razão entre os valores que correspondem ao 
>lucro e ao preço de custo desse artigo pode ser expressa pela 
>fração: 
> 
> a)1/4 b)1/5 c)4/5 d)5/6 e)6/5 
> 
>-- 

_
Voce quer um iGMail protegido contra vírus e spams? 
Clique aqui: http://www.igmailseguro.ig.com.br



[obm-l] Olimpíada brasileira

2004-03-17 Por tôpico Fabio Henrique
A Olimpíada Brasileira de Matemática para ensino médio e fundamental já tem 
data? Como posso fazer para inscrever os alunos do colégio onde trabalho? 
Obrigado. 
Fabio Henrique 

_
Voce quer um iGMail protegido contra vírus e spams? 
Clique aqui: http://www.igmailseguro.ig.com.br



Re: [obm-l] Dica sobre livro

2004-03-17 Por tôpico Fabio Henrique
Gosto do livro da Diomara e Maria Cândida. 


Em 16 Mar 2004, [EMAIL PROTECTED] escreveu: 

>Olá pessoal, gostaria de uma dica sobre livros de vocês. 
> 
>Eu usei o Hamilton Guidorizzi e o Louis Leithold para 
>as disciplinas Cálculo I e II, porém para Cálculo III 
>achei meio ruim a didática destes livros. Pesquisei no 
>Geraldo Ávila, mais nao gostei, alguem sabe outro autor 
>em portugues principalmente na área funçoes vetorias e 
>f. reais de varias variaveis reais? 
> 
>Atenciosamente, 
> 
>Futuro Engenheiro Eletricista 
>Osvaldo Mello Sponquiado FEIS - UNESP 
>Usuário em GNU/Linux 
> 
>__ 
>Acabe com aquelas janelinhas que pulam na sua tela. 
>AntiPop-up UOL - É grátis! 
>http://antipopup.uol.com.br/ 
> 
>= 
>Instruções para entrar na lista, sair da lista e usar a lista em 
>http://www.mat.puc-rio.br/~nicolau/olimp/obm-l.html 
>= 
> 
>-- 

_
Voce quer um iGMail protegido contra vírus e spams? 
Clique aqui: http://www.igmailseguro.ig.com.br



[obm-l] Re: Res: [off-topic] Pentagono regular

2004-03-14 Por tôpico Fabio Henrique
Parece-me que a indignação do Rafael dá-se contra os pareceres pouco 
esclarecedores do Dirichlet. 

Em 14 Mar 2004, [EMAIL PROTECTED] escreveu: 

>On Sun, Mar 14, 2004 at 05:35:46PM -0300, Rafael wrote: 
>> Concordo em absoluto com tudo o que você expôs, Artur. Discordo 
>> veementemente de outra coisa: apelar para a Trigonometria quando não se 
>> enxerga algo mais simples em Geometria. Geralmente, as soluções 
>> trigonométricas nada têm de elegantes e, para muitos casos, embora 
>resolvam 
>> o problema, dão um trabalho muito maior do que se utilizássemos conceitos 
>de 
>> Geometria. A verdade é que a Geometria exige uma percepção de construções 
>> que a Trigonometria, de certa forma, dispensa. 
> 
>Peço a todos um pouco mais de calma ao discutir algo que é puro gosto: 
>o Dirichlet gosta de soluções trigonométricas e o Rafael não. 
>Tudo bem que cada um queira dar a sua opinião neste debate milenar, 
>mas sem esquentar demais. 
> 
>A minha impressão como professor é que existem claras diferenças entre 
>indivíduos: uns tem mais facilidade para um tipo de solução, outros 
>tem mais facilidade com outro tipo. O bom estudante de olimpíada conhece 
>os seus próprios pontos fortes e fracos e sabe qual a melhor estratégia 
>para ele. Ele também sabe que não deve desprezar os métodos que não 
>são especialmente do gosto dele. 
> 
>[]s, N. 
>= 
>Instruções para entrar na lista, sair da lista e usar a lista em 
>http://www.mat.puc-rio.br/~nicolau/olimp/obm-l.html 
>= 
> 
>-- 

_
Voce quer um iGMail protegido contra vírus e spams? 
Clique aqui: http://www.igmailseguro.ig.com.br



Re: [obm-l] Livros de quimica e fisica para provas do ITA/IME

2004-03-14 Por tôpico Fabio Henrique
Eu sugeriria os do Feltre com o SETSUO YOSHINAGA. 

Você só vai encontrar em lojas de livros usados. 



Em 13 Mar 2004, [EMAIL PROTECTED] escreveu: 

>Apesar dessa não ser a lista ideal para fazer estas perguntas foi a que 
>mais se aproximou, então la vai: 
>Quais são os livros de quimica e fisica mais indicado para quem vai 
>prestar ITA e IME? 
>Atenciosamente, 
>Allan 
> 
>= 
>Instruções para entrar na lista, sair da lista e usar a lista em 
>http://www.mat.puc-rio.br/~nicolau/olimp/obm-l.html 
>= 
> 
>-- 

_
Voce quer um iGMail protegido contra vírus e spams? 
Clique aqui: http://www.igmailseguro.ig.com.br



Re: [obm-l] En: Putnam Question

2004-03-14 Por tôpico Fabio Henrique
Encontrei (EC.TS.TR)/(BC.AE.TC). 
Confere? 


Em 12 Mar 2004, [EMAIL PROTECTED] escreveu: 

>Sauda,c~oes, 
> 
>Um de geometria de uma outra lista. 
> 
>[]'s 
>Luis 
> 
>-Mensagem Original- 
>De: "Ben" <@hotmail.com> 
>Para: 
>Enviada em: segunda-feira, 9 de fevereiro de 2004 17:31 
>Assunto: Putnam Question 
> 
>> This is a problem I found in the 2001 Putnam exam. Anyone want to take 
>> a stab at it? 
>> 
>> Triangle ABC has an area 1. Points E, F, and G,lie respectively on 
>> sides BC , CA, and AB, such that AE bisects BF at point R, BF bisects 
>> CG at point S, and CG bisects AE at point T, Find the area of triangle 
>> RST. 
>> 
>> And go 
>> 
> 
>= 
>Instruções para entrar na lista, sair da lista e usar a lista em 
>http://www.mat.puc-rio.br/~nicolau/olimp/obm-l.html 
>= 
> 
>-- 

_
Voce quer um iGMail protegido contra vírus e spams? 
Clique aqui: http://www.igmailseguro.ig.com.br



[obm-l] Re: Seqüência

2004-03-08 Por tôpico Fabio Henrique
312211 
e 
13112221 



Em 8 Mar 2004, [EMAIL PROTECTED] escreveu: 

>Eis um problema muito interessante: 
> 
>Seja a seqüência numérica (1, 11, 21, 1211, 111221, ...), quais são o sexto 
>e sétimo termos? 
> 
>Abraços, 
> 
>Rafael de A. Sampaio 
> 
>= 
>Instruções para entrar na lista, sair da lista e usar a lista em 
>http://www.mat.puc-rio.br/~nicolau/olimp/obm-l.html 
>= 
> 
>-- 

_
Voce quer um iGMail protegido contra vírus e spams? 
Clique aqui: http://www.igmailseguro.ig.com.br



[obm-l] Re: ú]¾'Z

2004-03-07 Por tôpico Fabio Henrique
Já que você perguntou, se não estou enganado, alguém falou que a bandeira do 
Brasil tem DOIS retângulos, um losango e um círculo. 
??? 


 Em 07 Mar 2004, [EMAIL PROTECTED] escreveu: 

>Re: [obm-l] dúvida 
> 
>on 07.03.04 16:35, TSD at [EMAIL PROTECTED] wrote: 
> 
> OLÁ AMIGOS ESTOU COM UMA PEQUENA DÚVIDA. DE QUANTAS MANEIRAS EU POSSO 
COLORIR A BANDEIRA DO BRASIL DE MODO QUE AS REGIÕES COMUNS(ÁREA DELIMITIDA 
PELAS FIGURAS) SEJAM PINTADAS COM CORES DIFERENTE. 
> 
>AGUARDO RESPOSTA 
> 
>Esse problema tah mal definido. E as estrelas? Tem que ser pintadas tambem? 
> 
>E a inscricao ORDEM E PROGRESSO? Pode ser cada letra duma cor? 
> 
>-- 

_
Voce quer um iGMail protegido contra vírus e spams? 
Clique aqui: http://www.igmailseguro.ig.com.br



[obm-l] Tri isósceles

2004-02-29 Por tôpico Fabio Henrique
Problema do triângulo isósceles 

Vamos imaginar que A=20º, AB=AC=1. Considere E sobre AC e F sobre AB. 
CBE=60º e BCF=50º 
O ângulo procurado é BEF 
De saída, sabemos que: 
EBF=20º 
FCE=30º 
BFC=50º 
BEC=40º 
Desconfiando que os triângulos AFC e BFE fossem semelhantes, resolvi tentar 
provar o fato. 

Aplicando Lei dos Senos no triângulo ABC temos BC=sen20/sen80 
Aplicando Lei dos Senos no triângulo ABE temos BE=sen20/sen40 
Aplicando Lei dos Senos no triângulo ABC temos AF=sen30/sen50 
Além disso, BF=1-(sen30/sen50) 

AF/AC=sen30/sen50 

BF/BE=[1-(sen30/sen50)]/(sen20/sen40) 
BF/BE=[(sen50-sen30)/sen50)]/(sen20/sen40) 

Neste ponto, precisamos observar que sen50=sen40.cos10+sen10.cos40 
   e sen10=sen40.cos10-sen10.cos40 
Logo, sen50-sen10=2.sen10.cos40 
Então BF/BE=[2.sen10.cos40/sen50]/[sen20/sen40] 
BF/BE=2.sen10.cos40.sen40/sen50.sen20 
BF/BE=sen10.sen80/sen50.sen20 
BF/BE=sen10.cos10/sen50.sen20 
BF/BE=sen20/2.sen50.sen20 
BF/BE=0,5/sen50 
BF/BE=sen30/sen50=AF/AC 

Isto garante que os triângulos AFC e BFE são semelhantes. 
Assim med(ACF)=med(BEF)=30º 

SENHORES, DÊEM UMA OLHADA E CONFIRAM SE ME EQUIVOQUEI EM ALGUMA PASSAGEM. 

[]'s 

Fabio Henrique. 

_
Voce quer um iGMail protegido contra vírus e spams? 
Clique aqui: http://www.igmailseguro.ig.com.br



Re: [obm-l] Ops! Problema de quadrado perfeito

2004-02-28 Por tôpico Fabio Henrique
Claro! Eu dei uma mongolizada incrível! 

Veja só. Quando quebrei o 99, o fiz de uma só maneira: 9 x 11. 
No entanto, isto pode ser feito de TRÊS modos: 9x11, 3x33 e o trivial 1x99. 
A primeira nós já fizemos. Vamos aos outros: 
2º) p+k=33 e p-k=3 => p=18 e k=15 (O valor de x é 225) 
3º) p+k=99 e p-k=1 => p=50 e k=49 (O valor de x é 2401). 

Desculpe-me o equívoco. 




Em 28 Feb 2004, [EMAIL PROTECTED] escreveu: 

>Opa... tipo, entendi mais ou menos seu raciocinio.. 
> 
>mas o gabarito é 3. eu tbm tinha achado 1... mas errei. 
> 
>hmm ... 
>- Original Message - 
>From: "Fabio Henrique" 
>To: 
>Sent: Saturday, February 28, 2004 11:00 AM 
>Subject: Re: [obm-l] Problema de quadrado perfeito 
> 
>> Seja x = k^2 e x+99 = p^2 
>> Desta forma, k^2 +99 = p^2 
>> p^2 - k^2 = 9 x 11 
>> (p-k)(p+k)= 9 x 11 
>> Assim, p=10 e k=1 ou p=-10 e k=-1 
>> Logo, x=1. 
>> 
>> Em 28 Feb 2004, [EMAIL PROTECTED] escreveu: 
>> 
>> >1 ) Quantos inteiros positivos x são 
>> >tais que tanto x quanto x+ 99 são quadrados perfeitos? 
>> > 
>> > Eu nao entendi bem o enunciado.. quem puder da uma 
>> >explicada ae eu agradeço! 
>> > 
>> > Um abraço!!! 
>> > 
>> >-- 
>> 
>> _ 
>> Voce quer um iGMail protegido contra vírus e spams? 
>> Clique aqui: http://www.igmailseguro.ig.com.br 
>> 
>> 
>> Esta mensagem foi verificada pelo E-mail Protegido Terra. 
>> Scan engine: VirusScan / Atualizado em 27/02/2004 / Versão: 1.4.1 
>> Proteja o seu e-mail Terra: http://www.emailprotegido.terra.com.br/ 
>> 
>> E-mail classificado pelo Identificador de Spam Inteligente Terra. 
>> Para alterar a categoria classificada, visite 
>> 
>http://www.terra.com.br/centralunificada/emailprotegido/imail/imail.cgi?+_u=fabiocontreiras&_l=1077977988.52812.13512.turvo.terra.com.br
> 
>> 
> 
>= 
>Instruções para entrar na lista, sair da lista e usar a lista em 
>http://www.mat.puc-rio.br/~nicolau/olimp/obm-l.html 
>= 
> 
>-- 

_
Voce quer um iGMail protegido contra vírus e spams? 
Clique aqui: http://www.igmailseguro.ig.com.br



Re: [obm-l] Diedros e Triedros

2004-02-28 Por tôpico Fabio Henrique
"A soma das medidas (em graus) das faces de um triedro qualquer é menor que 
360º." 

0º < A + B + C < 360º 

Considere a semi-reta Va’ oposta a Va. No triedro V(a’,b,c) temos bc < ba’+ 
ca’. 
Observe que ab + ba’ = 180 e ac + ca’ = 180 
Então, ab + ba’ +  ac + ca’ =  360 
Como bc < ba’+ ca’, ta provado. 

Outro abraço. 
Fabio Henrique. 



Em 27 Feb 2004, [EMAIL PROTECTED] escreveu: 

>Boa noite aos colegas da lista. 
> 
>Há pouco tempo, estava eu estudando diedros, triedros, poliedros e ângulos 
>poliédricos. O livro que estava lendo afirmava verdadeiras algumas 
>desigualdades, mas não as demonstrava. Assim, fiquei sem saber se 
>realmente elas são "intuitivamente" verificadas somente ou se existe uma 
>demonstração formal, que ainda não encontrei. 
> 
>Sejam A, B e C faces e d_1, d_2 e d_3 diedros (ângulos entre faces), as 
>afirmações são: 
> 
>"A soma das medidas (em graus) das faces de um triedro qualquer é menor que 
>360º." 
> 
>0º < A + B + C < 360º 
> 
>"Em todo triedro, qualquer face é menor que a soma das outras duas." 
> 
>|B - C| < A < B + C 
> 
>"A soma dos diedros de um triedro está compreendida entre 2 retos e 6 
>retos." 
> 
>180º < d_1 + d_2 + d_3 < 540º 
> 
>"Em qualquer triedro, a medida (em graus) de um diedro aumentada de 180º 
>supera a soma das medidas dos outros dois." 
> 
>d_1 + 180º > d_2 + d_3 
> 
>Fica o meu agradecimento desde já a quem puder demonstrar, comentar ou 
>quaisquer referências que possam ser consultadas sobre o assunto. 
> 
>Abraços, 
> 
>Rafael de A. Sampaio 
> 
>= 
>Instruções para entrar na lista, sair da lista e usar a lista em 
>http://www.mat.puc-rio.br/~nicolau/olimp/obm-l.html 
>= 
> 
>-- 

_
Voce quer um iGMail protegido contra vírus e spams? 
Clique aqui: http://www.igmailseguro.ig.com.br



Re: [obm-l] Diedros e Triedros

2004-02-28 Por tôpico Fabio Henrique
"Em todo triedro, qualquer face é menor que a soma das outras duas." 
|B - C| < A < B + C 

Seja um triedro v(a,b,c). Vamos supor que ac é a maior face(ou ângulo de 
face) de V. 

Construa em ac um ângulo b'c congruente a bc. 
Tome um ponto B'em b' de forma que o segmento VB seja congruente ao segmento 
VB'. 
Considere uma seção ABC (A em VA). 
Os triângulos BVB e B'VC são congruentes. Conclui-se que BC congr. a B'C. 
No triângulo ABC, temos AC AB'+B'C AB'Boa noite aos colegas da lista. 
> 
>Há pouco tempo, estava eu estudando diedros, triedros, poliedros e ângulos 
>poliédricos. O livro que estava lendo afirmava verdadeiras algumas 
>desigualdades, mas não as demonstrava. Assim, fiquei sem saber se 
>realmente elas são "intuitivamente" verificadas somente ou se existe uma 
>demonstração formal, que ainda não encontrei. 
> 
>Sejam A, B e C faces e d_1, d_2 e d_3 diedros (ângulos entre faces), as 
>afirmações são: 
> 
>"A soma das medidas (em graus) das faces de um triedro qualquer é menor que 
>360º." 
> 
>0º < A + B + C < 360º 
> 
>"Em todo triedro, qualquer face é menor que a soma das outras duas." 
> 
>|B - C| < A < B + C 
> 
>"A soma dos diedros de um triedro está compreendida entre 2 retos e 6 
>retos." 
> 
>180º < d_1 + d_2 + d_3 < 540º 
> 
>"Em qualquer triedro, a medida (em graus) de um diedro aumentada de 180º 
>supera a soma das medidas dos outros dois." 
> 
>d_1 + 180º > d_2 + d_3 
> 
>Fica o meu agradecimento desde já a quem puder demonstrar, comentar ou 
>quaisquer referências que possam ser consultadas sobre o assunto. 
> 
>Abraços, 
> 
>Rafael de A. Sampaio 
> 
>= 
>Instruções para entrar na lista, sair da lista e usar a lista em 
>http://www.mat.puc-rio.br/~nicolau/olimp/obm-l.html 
>= 
> 
>-- 

_
Voce quer um iGMail protegido contra vírus e spams? 
Clique aqui: http://www.igmailseguro.ig.com.br



Re: [obm-l] Problema de quadrado perfeito

2004-02-28 Por tôpico Fabio Henrique
Seja x = k^2 e x+99 = p^2 
Desta forma, k^2 +99 = p^2 
 p^2 - k^2 = 9 x 11 
 (p-k)(p+k)= 9 x 11 
Assim, p=10 e k=1 ou p=-10 e k=-1 
Logo, x=1. 

Em 28 Feb 2004, [EMAIL PROTECTED] escreveu: 

>1 ) Quantos inteiros positivos x são 
>tais que tanto x quanto x+ 99 são quadrados perfeitos? 
> 
> Eu nao entendi bem o enunciado.. quem puder da uma 
>explicada ae eu agradeço! 
> 
> Um abraço!!! 
> 
>-- 

_
Voce quer um iGMail protegido contra vírus e spams? 
Clique aqui: http://www.igmailseguro.ig.com.br



Re: RE: [obm-l]

2004-02-25 Por tôpico Fabio Henrique
PRIMEIRA QUESTÃO: 

Vamos tentar "arrumar" a expressão x^2+3xy+y^2 dentro de alguma outra que 
tenha vindo diretamente de 2x+y. 

2x+y=1 
(2x+y)^2=1 
(2x+y)^2=4x^2+4xy+y^2=(x^2+3xy+y^2)+3x^2+xy=1 
(2x+y)^2=(x^2+3xy+y^2)+x.(3x+y)=1 
(2x+y)^2=(x^2+3xy+y^2)+x.(x+2x+y)=1 
(2x+y)^2=(x^2+3xy+y^2)+x.(x+1)=1 
(x^2+3xy+y^2)=1-x-x^2 
Calcule agora o maior valor de -x^2-x+1 (use -(delta)/4a) 


24 Feb 2004, [EMAIL PROTECTED] escreveu: 

>Eu acho que consegui a terceira. 
> 
> (ab)^2 = b^2 + 10ab + 10ab + 100a^2 (I) 
> (ba)^2 = a^2 + 10ab + 10ab + 100b^2 (II) 
> 
> fazendo (I) - (II) -> 99a^2 - 99b^2 
> 
>(cc)^2= c^2 + 20c^2 +100c^2 
> 
>99a^2 - 99b^2 = 121c^2 -> 9a^2-9b^2 = 11c^2 
> 
> 9(a^2-b^2) = 11c^2 -> como 11 eh primo, temos c^2 múltiplo de nove, 
>ou c=0,3,6 ou 9 
> 
> fazendo c=3, 9(a^2-b^2) = 11.9 -> a^2-b^2 = 11 
> 
> por tentativa e erro temos a=6 e b = 5 
> 
> 65^2 - 56^2 = 33^2 
> 
>>OLÁ AMIGOS ESTOU COM MAIS ALGUMAS DÚVIDAS. PODERIAM AJUDAR POR FAVOR. 
>> 
>>1) Se 2x + y = 1, com x e y reais, então o maior valor da expressão x² 
>>+ 3xy + y² é igual a ; 
>>A)5/4 B)7/4 C)13/8 D)17/8 E)31/16 
>> 
>>2) Se x e y são números inteiros e positivos, representa-se o máximo 
>>divisor comum de x e y por mdc (x,y); assim, o número de pares ordenados 
>>(x,y) que são soluções do sistema : 
>> x + y = 810 
>>mdc(x,y)E 
>> 
>>A)6 B)8 C)10 D)16 E)18 
>> 
>>3) Se a, b e c são algarismos distintos, no sistema de numeração 
>>decimal existe um único número de dois algarismos (ab) tal que (ab)² - 
>>(ba)² = (cc)². 
>>O valor de (a + b + c) é igual a: 
>>A) 11 B)12 C)13 D)14 E)15 
>> 
> 
>_ 
>MSN Messenger: converse com os seus amigos online. 
>http://messenger.msn.com.br 
> 
>Instruções 
>para entrar na lista, sair da lista e usar a lista em 
>http://www.mat.puc-rio.br/~nicolau/olimp/obm-l.html 
> 
> 
>-- 

_
Voce quer um iGMail protegido contra vírus e spams? 
Clique aqui: http://www.igmailseguro.ig.com.br



[obm-l] Re: dúvidass

2004-02-23 Por tôpico Fabio Henrique
Em 21 Feb 2004, [EMAIL PROTECTED] escreveu: 

>olá amigos poderiam ajudar neste problema; 
> 
> 1) Por um ponto M exterior a um círculo de centro 
> O traçam-se as tangentes MA e MB(são segmentos). se a corda 
>AB(segmento) é um lado do pentágono regular inscrito nesse círculo, a 
medida do 
>ângulo AMB(M^) é igual a : 
> a)144º b)120º c)108º d)96º e)72º 
> 
O menor arco AB vale 360/5=72. Logo, o maior arco AB vale 288. 
O ângulo excêntrico esterior AMB vale (288-72)/2 = 108 


> 2) coloquem em ordem crescente os números 
>abaixos; 
> 
> sqtr(3) , sqtr5, raizcúbica de 2, raiz quinta de 3, raiz 
>quinta de 5. 
> 
> obs: não usar calculadora em hipótese alguma. 
> 
> como faço essa! 

Vou usar a simbologia arb como raiz a-ézima de b. 

2r3 = 30r(3^15) 
2r5 = 30r(5^15) 
3r2 = 30r(2^10) 
5r3 = 30r(3^6) 
5r5 = 30r(5^6) 

Por enquanto sabemos que 
3^6 < 5^6 < 5^15 
onde estarao 3^15 e 2^10? 

Como 5^2 < 3^3 e, consequentemente 5^6 < 3^9, podemos perceber que 5^6 < 
3^15 que, por sua vez 'e menor do que 5^15. 

Assim: 3^6 < 5^6 < 3^15 < 5^15 

Posto que 32 < 125, logo 2^5 < 5^3 => 2^10 < 5^6. 
De forma an'aloga, 27 < 32 => 3^3 < 2^5 => 3^6 < 2^10 

Conclus~ao: 3^6 < 2^10 < 5^6 < 3^15 < 5^15 


> ajudem por favor. 
> 
>-- 

_
Voce quer um iGMail protegido contra vírus e spams? 
Clique aqui: http://www.igmailseguro.ig.com.br



[no subject]

2004-02-23 Por tôpico Fabio Henrique
SENHORES, O QUE HÁ COM VOCÊS? 

A segunda progressão não é uma progressão geométrica. 

2 + 1 +1/2 +... = 3 ? 

Errar é normal mas vamos ter mais cuidado. 

[]'s 
Fabio Henrique. 

Junior, 

1 + 2/2 + 3/4 + 4/8 + 5/16 + ... <=> 1 + 1 + 3/4 + 1/2 + 5/16 + ... <=> 1 + 
(1 + 1/2 + ...) + (3/4 + 5/16 + ...) 

Calculando o limite da soma para a primeira progressão, sabendo-se que a 
razão é 1/2 e o primeiro termo é 1: 

1/(1-1/2) = 2 

Calculando o limite da soma para a segunda progressão, sabendo-se que a 
razão é 5/16 / 3/4 = 5/12 e o primeiro termo é 3/4: 

3/4/(1-5/12) = 9/7 

Somando-se: 1 + 2 + 9/7 = 30/7 é o limite da soma dos infinitos termos para 
a P.G. inicial. 



faz o seguinte: 

1 = 1 
2/2 = 1/2 +1/2 
3/4 = 1/4 + 1/4 +1/4 

e assim sucessivamente 

soma coluna por coluna, 
então vc vai ter q 

1+2/2+3/4+4/8+5/16... = (1+1/2+...)+ (1/2 +1/4+..) + 
(1/4+ 1/8+...) +... 

= 2 + 1 +1/2 +... = 3 

_
Voce quer um iGMail protegido contra vírus e spams? 
Clique aqui: http://www.igmailseguro.ig.com.br



Re: [obm-l] duvidazinha

2004-02-23 Por tôpico Fabio Henrique
D=d.q+r 
3x^2 + 4 = x.3x + x-1 
3x^2 + 4 = 3x^2 + x-1 
4 = x-1 
x = 5 



Em 22 Feb 2004, [EMAIL PROTECTED] escreveu: 

>\OLÁ COLEGAS , PODERIAM AJUDAR NESTE PROBLEMA: 
> 
> 1) numa divisão, o dividendo é igual a 3x²+4, o divisor é 
>igual a x, o quociente é o triplo do divisor e o resto é o maior possível. 
O 
>número que corresponde `a soma do dividendo com o resto é igual a? 
> 
>-- 

_
Voce quer um iGMail protegido contra vírus e spams? 
Clique aqui: http://www.igmailseguro.ig.com.br



Re: [obm-l] torres

2004-02-20 Por tôpico Fabio Henrique
Pode pensar com as rainhas... 


Em 20 Feb 2004, [EMAIL PROTECTED] escreveu: 

>Ola, 
> 
>Tem certeza que digitou corretamente o enunciado ? 
> 
>Seria: 
> 
>De forma que 2 torres nao estejam na mesma linha 
> 
>OU 
> 
>De forma que as 8 torres nao se ataquem ? 
> 
>Ps: Eu ja vi um bem interessante: Coloque 8 rainhas em um tabuleiro sem que 
nenhuma ataque as outras  
> 
>Em uma mensagem de 19/2/2004 17:33:00 Hora padrão leste da Am. Sul, 
[EMAIL PROTECTED] escreveu: 
> 
>De quantas maneiras podemos arrumar 8 torres iguais em um tabuleiro de 
forma 
> 
>que duas torres não estejam na mesma linha, coluna ou diagonal? 
> 
>-- 

_
Voce quer um iGMail protegido contra vírus e spams? 
Clique aqui: http://www.igmailseguro.ig.com.br



[obm-l] torres

2004-02-19 Por tôpico Fabio Henrique
De quantas maneiras podemos arrumar 8 torres iguais em um tabuleiro de forma 
que duas torres não estejam na mesma linha, coluna ou diagonal? 

_
Voce quer um iGMail protegido contra vírus e spams? 
Clique aqui: http://www.igmailseguro.ig.com.br



Re: [obm-l] Orgia de livros

2004-02-19 Por tôpico Fabio Henrique
Só não acho interessante vincular à nossa lista algo ilegal. Façam por fora. 
Não nos envolvam. 


Em 19 Feb 2004, [EMAIL PROTECTED] escreveu: 

>Gostei do link. Muito bom. 
> Se a pessoa está ou não fora da lei eu não 
>sei, mas não me importo hahaha... 
> Mas o fato é que eu estou ávido de conhecimento como 
>Tartaglia (que furtou livros para aprender álgebra). 
> e já baixei tudo pra uma pasta no 
>meu computador (fiz isso antes da página sair do ar). 
> 
> VALEU BROTHER 
> 
>[]s Ronaldo L. Alonso 
> 
>>Parece que esta página é ilegal... 
>> 
>>Muitos livros colocados lá tem os seus direitos protegidos, como o 
Concrete 
>>Mathematics. Outros foram temporariamente disponibilizados na internet por 
>>seus autores, como os do Wilf. 
>> 
>>Não sei quem mantém a página, mas perante as leis de direitos autorais, 
>esta 
>>pessoa está cometendo um crime. 
>> 
>>Abraços, Paulo 
>> 
> 
>_ 
>Voce quer um iGMail protegido contra vírus e spams? 
>Clique aqui: http://www.igmailseguro.ig.com.br 
>Ofertas imperdíveis! Link: http://www.americanas.com.br/ig/ 
> 
>= 
>Instruções para entrar na lista, sair da lista e usar a lista em 
>http://www.mat.puc-rio.br/~nicolau/olimp/obm-l.html 
>= 
> 
>-- 

_
Voce quer um iGMail protegido contra vírus e spams? 
Clique aqui: http://www.igmailseguro.ig.com.br



Re: [obm-l] Ajuda

2004-02-15 Por tôpico Fabio Henrique
Se você pretende estudar problemas mais profundos e demonstrativos, vale a 
pena o do Wagner. Outro que gosto é o do Timótheo Pereira, mas este vai ser 
difícil achar. 


Em 15 Feb 2004, [EMAIL PROTECTED] escreveu: 

>Ficar bom em Geometria! 
> 
>- Original Message - 
>From: Fabio Henrique 
>To: 
>Sent: Sunday, February 15, 2004 9:56 AM 
>Subject: Re: [obm-l] Ajuda 
> 
>> Depende do teu propósito. 
>> 
>> 
>> Em 14 Feb 2004, [EMAIL PROTECTED] escreveu: 
>> 
>> >Gostaria da ajuda dos amigos. 
>> > 
>> > Qual destes dois livros de geometria plana é 
>> >melhor? 
>> > Os livros de geometria do Morgado e do Eduardo 
>> >Wagner(Geometria1 e 2) 
>> > 
>> > Ou o Fundamentos da Matemática Volume 
>> >9 
>> > 
>> >-- 
>> 
>> _ 
>> Voce quer um iGMail protegido contra vírus e spams? 
>> Clique aqui: http://www.igmailseguro.ig.com.br 
>> 
>> 
>= 
>Instruções para entrar na lista, sair da lista e usar a lista em 
>http://www.mat.puc-rio.br/~nicolau/olimp/obm-l.html 
>= 
> 
>-- 

_
Voce quer um iGMail protegido contra vírus e spams? 
Clique aqui: http://www.igmailseguro.ig.com.br



Re: [obm-l] Compras

2004-02-15 Por tôpico Fabio Henrique
A moto custa, à vista, 8000.(1-x) e este é o valor de que dispõe Paulo. 
Deste valor, Paulo tira 4000 para pagar a 1ª. O restante [8000.(1-x)-4000] 
será aplicado. Após 1 mês, Paulo terá [8000.(1-x)-4000]. 1,25 que deve ser 
igual à 2ª prestação. 
Então, [8000.(1-x)-4000].1,25 = 4000 => x=0,1 


Em 15 Feb 2004, [EMAIL PROTECTED] escreveu: 

>Uma motocicleta cujo preço de 
>tabela é R$ 8.000,00 é vendida , à vista , com desconto de x% ou 
>em duas parcelas iguais 
> de R$ 4.000,00 , sendo a 
>primeira no ato da compra e segunda um mês após a compra. Supondo 
>que o Sr. Paulo dispõe 
> do dinheiro necessário 
>para pagar à vista e que ele sabe que a diferença entre o preço à vista e 
>a primeira parcela pode ser 
> aplicada no mercado financeiro a 
>uma taxa de 25% ao mês. Nessas condições , qual é o valor de x que torna 
>indiferente 
> comprar à vista ou a prazo 
>: 
> 
> a) 
>10 
>b)12 c) 
>15 D) 16 
> 
> -- 
> Esta mensagem foi verificada pelo sistema de antivírus e 
> acredita-se estar livre de perigo. 
> 
>-- 

_
Voce quer um iGMail protegido contra vírus e spams? 
Clique aqui: http://www.igmailseguro.ig.com.br



Re: [obm-l] Ajuda

2004-02-15 Por tôpico Fabio Henrique
Depende do teu propósito. 


Em 14 Feb 2004, [EMAIL PROTECTED] escreveu: 

>Gostaria da ajuda dos amigos. 
> 
> Qual destes dois livros de geometria plana é 
>melhor? 
> Os livros de geometria do Morgado e do Eduardo 
>Wagner(Geometria1 e 2) 
> 
> Ou o Fundamentos da Matemática Volume 
>9 
> 
>-- 

_
Voce quer um iGMail protegido contra vírus e spams? 
Clique aqui: http://www.igmailseguro.ig.com.br



Re: [obm-l] OI

2004-02-14 Por tôpico Fabio Henrique
Em 14 Feb 2004, [EMAIL PROTECTED] escreveu: 

>OLA AMIGOS PODERIA AJUDAR NESTES 
>QUESTÕES. 
> 
>1) Para se demarcar o 
>estacionamento de todo o lado direito de uma rua reta, foram pintados 20 
>retângulos de 4,5metros de comprimento e 2,5 metros de largura. Sabendo-se 
que 
>os carros estacionam no sentido do comprimento dosretângulos e da rua, e à 
>frente e atrás de cada um dos retângulos tem 50 centímetros de folga, qual 
é o 
>comprimento, emmetros, da rua? 
> 
>A) 90 B) 
> 90,5 C) 95 D) 100 E) 
> 100,5 
> 
>2) Uma massa 
>fermentada, ao ser colocada para descansar, ocupou uma área circular S de 
raio 
>r. Após um certo tempo t,ela passou a ocupar uma área 21% maior que S. Qual 
o 
>valor de r, em centímetros, para que a massa não transborde,quando colocada 
para 
>descansar durante o tempo t, em um tabuleiro circular de raio 22 
>centímetros? 
> 

[[se a área aumenta 21% em um tempo t, neste mesmo tempo, o raio aumenta 
10%. Então a resposta é 20 cm, para que o raio aumente 10% e chegue a 22 
cm.]] 


>3) Um bebedouro que usa 
>garrafão de água tem 2,5 metros de serpentina por onde a água passa para 
gelar. 
>Sabe-se quetal serpentina gasta 12 segundos para ficar totalmente gelada. 
>Colocando-se um garrafão de 10 litros e ligando-se obebedouro, leva-se 5 
minutos 
>para que toda a água saia gelada. Se nas mesmas condições, fosse colocado 
um 
>garrafãode 20 litros no lugar do de 10 litros, o tempo gasto para que toda 
a 
>água saísse gelada seria de: 
> 
>A) 9 minutos e 36 segundos. 
> 
>B) 9 minutos e 48 segundos. 
> 
>C) 10 minutos. 
> 
>D) 10 minutos e 12 segundos. 
> 
>E) 11 minutos. 
> 
>-- 
[[Dos 5 minutos, 12 segundos são para gelar a serpentina. Logo, para gelar o 
conteúdo do garrafão (10 litros), precisa-se de 4 min e 48 segundos = 4 + 
4/5 min = 24/5 min, sendo 24/50 min para cada litro. Para 20 litros temos 20 
x 24/50 min = 48/5 min = 9 + 3/5 min = 9 min 36 seg. Adicione os 12 seg da 
serpentina. Resp: B 

_
Voce quer um iGMail protegido contra vírus e spams? 
Clique aqui: http://www.igmailseguro.ig.com.br



Re: RES: RES: [obm-l] area de triangulo

2004-02-11 Por tôpico Fabio Henrique
Não faltou dizer que as circunferências são tangentes entre si duas a duas? 

Em 10 Feb 2004, [EMAIL PROTECTED] escreveu: 

> 
> Bom, o 
>ângulo formado entre um lado do triangulo e um dos vértices do triangulo 
até o 
>centro da circunferência mais próxima desse vértice é 30°. Desse centro até 
o 
>lado são 4cm, pois ela é tangente. Como o ângulo é de 30° então do ponto de 
>tangência até o vértice do triangulo vai ser 4sqrt(3) cm. Isso obviamente 
vale 
>pro outro lado do triângulo. Logo pra descobrir o tamanho do lado falta só 
o 
>meio do lado que é um segmento de 8cm, formado pela união dos 
>centros das circunferências internas de raio 4cm. Logo o lado do triângulo 
vale 
>4sqrt(3) + 8 + 4sqrt(3) = 8(sqrt (3 ) +1) cm. 
> 
> Daí: 
> 
> A= 
>L²sqrt (3 )/4 
> 
> Desenvolvendo 
>dá 32[2sqrt(3) + 3] cm² 
> 
> Avisem-me 
>se por acaso saiu algo errado... Douglas Ribeiro 
> 
> -Mensagem original- 
> 
> De: [EMAIL PROTECTED] puc -rio. br [mailto:[EMAIL PROTECTED] 
> Em nome de [EMAIL PROTECTED] com 
> 
> Enviada em: terça-feira, 10 de 
>fevereiro de 2004 00:29 
> 
> Para: [EMAIL PROTECTED] puc -rio. br 
> 
> Assunto: [obm-l] area de triangulo 
> 
> Ola pessoal, 
> 
>Imaginem um triangulo equilatero com 3 circunferencias 
>de raio 4 cm inscritas neste triangulo. Cada lado do triangulo eh tangente 
a 2 circunferencia . Qual a area do triangulo 
>? 
> 
> Yahoo! 
>Mail - 6MB, anti-spam e antivírus gratuito. Crie 
>sua conta agora! 
> 
>-- 

_
Voce quer um iGMail protegido contra vírus e spams? 
Clique aqui: http://www.igmailseguro.ig.com.br



Re: [obm-l] RE: [obm-l] Iezzi dúvida

2004-01-28 Por tôpico Fabio Henrique
Achei mais fácil decompor o 5 em 10/2. 


Em 28 Jan 2004, [EMAIL PROTECTED] escreveu: 

>O leonardo está correto, apenas temos que decompor 100 em 2^2 . 5^2 , ai 
fica fácil vc decompoe tb a parte do 2 e do 5 , depois disso somente é um 
jogo algebrico 
> 
> até chegar em (2^2.5^2)^(x^2 - y) = (2^2.5^2) 
> 
> Pérsio 
> 
> Carlos Alberto wrote: 
> 
> Como chegou nesse resultado?? 
> 
> leonardo mattos wrote: 
> Ola, 
> 
>Antes de substituir desenvolva a equacao (I) e vc vera que (x^2-y)=1 
> 
>Um abraço, 
>Leonardo 
> 
>>From: Tâni Aparecida 
>>Reply-To: [EMAIL PROTECTED] 
>>To: [EMAIL PROTECTED] 
>>Subject: [obm-l] Iezzi dúvida 
>>Date: Wed, 28 Jan 2004 12:31:03 -0300 (ART) 
>> 
>>Resolva o seguinte sistema: 
>> 
>>2^(2.(x^2-y)) = 100 . 5^(2.(y-x^2)) ( I ) 
>>x + y = 5 ( II ) 
>> 
>> 
>>---
>> 
>> 
>>Comecei a resolver dessa maneira em ( II ) tenho que 
>>y = 5-x substituo em ( I ) que fica 
>> 
>>2^(2.(x^2+x-5)) = 100 . 5^(2.(-x^2 -x + 5)) 
>> 
>>e agora como faço para resolver isso? 
>> 
>> 
>> 
>>- 
>>Yahoo! Mail - 6! 
>! MB, 
> anti-spam e antivírus gratuito. Crie sua conta agora! 
> 
>_ 
>MSN Hotmail, o maior webmail do Brasil. http://www.hotmail.com 
> 
>= 
>Instruções para entrar na lista, sair da lista e usar a lista em 
>http://www.mat.puc-rio.br/~nicolau/olimp/obm-l.html 
>= 
> 
>-- 

_
Voce quer um iGMail protegido contra vírus e spams? 
Clique aqui: http://www.igmailseguro.ig.com.br



Re: [obm-l] Impossibilidade do movimento

2004-01-24 Por tôpico Fabio Henrique
Acho curioso que sempre que se toca no assunto "Paradoxo de Aquiles e a 
Tartaruga", de Zenon, sempre se recorre a somas infinitas como explicação do 
paradoxo. Mesmo quando o assunto foi questão da prova da UFRJ, o argumento 
usado foi o mesmo. 
Parece-me que a explicação do paradoxo é o fato de que este foi construído 
sobre condições idealizadas e não reais. Há um momento em que a distância 
entre Aquiles e a tartaruga seria tão pequena (segundo as parcelas da soma 
infinita) que chegaria a ser menor do que o pé da tartaruga. Nunca 
vi/ouvi/li ninguém argumentar que o paradoxo criado por Zenon considera 
tanto a tartaruga quanto Aquiles como objetos pontuais, sem dimensão. O que 
de fato contraria o nosso senso prático. 
Estaria eu pensando bobagem? 

24 Jan 2004, [EMAIL PROTECTED] escreveu: 

>Partindo desse princípio, pode-se dizer que a cada termo adicionado naquela 
>soma, o valor total aumenta. Por exemplo, se eu utilizar 10 termos eu tenho 
>um valor; se eu utilizar 100 termos eu tenho outro maior, e assim 
>sucessivamente. Desse modo, como a soma é infinita e possui estritamente 
>termos positivos, seu resultado deveria ser infinito. No entanto, pelos 
>conhecimentos atuais de matemática, isso não ocorre. Muito estranho! 
> 
>- Original Message - 
>From: "Frederico Reis Marques de Brito" 
>To: 
>Sent: Saturday, January 24, 2004 9:47 AM 
>Subject: Re: [obm-l] Impossibilidade do movimento 
> 
>> Isto é absolutamente falso. Observe que 1/(10^n) tende a 0 quando 
>n 
>> tender a infinito, de forma estritamente decrescente, isto é , se n > m 
>=> 
>> 1/(10^n) < 1/(10^m), mas 0 não é um termo dessa sequência. Posto isto , é 
>> fácil ver que não existe um menor número e que as demais parcelas são 
>> múltiplas desta... 
>> 
>> Frederico. 
>> 
>> 
>> >From: "Marcelo Augusto Pereira" 
>> >Reply-To: [EMAIL PROTECTED] 
>> >To: 
>> >Subject: Re: [obm-l] Impossibilidade do movimento 
>> >Date: Fri, 23 Jan 2004 22:10:01 -0200 
>> > 
>> >O fato de essa soma ser calculável(1/9) não indica que existe um número 
>de 
>> >valor muito pequeno e que esse número seria o valor mínimo que possa 
>> >existir? Assim todos os outros números seriam múltiplos desse menor 
valor 
>> >possível, ou seja, esse número seria algo como um valor quântico. Dessa 
>> >forma, também existiria uma unidade quântica de deslocamento linear, o 
>que 
>> >faria com que a quantidade de pontos em um segmento de reta não fosse 
>> >infinita e o movimento fosse possível. Se para cada número existisse um 
>> >menor, a soma teria que ser infinita, e o resultado infinito. 
>> > 
>> >- Original Message - 
>> >From: "Frederico Reis Marques de Brito" 
>> >To: 
>> >Sent: Friday, January 23, 2004 9:27 PM 
>> >Subject: RE: [obm-l] Impossibilidade do movimento 
>> > 
>> > 
>> > > 
>> > > Essencialmente esse problema é ujm dos paradoxos de Zenão, um grego 
>> >antigo 
>> > > que usava a idéia de infinito para chegar a conclusões aparentemente 
>> > > absurdas, tais como a impossibilidade do movimento, por exemplo. 
Agora 
>> >vou 
>> > > dar uma de Dirichlet, o da lista é claro: Pense no seguinte, uma soma 
>de 
>> > > infinitas parcelas positivas é sempre infinito, ou não 
>necessariamente? 
>> >Para 
>> > > ajudar nessa resposta, pense em calcular, por exemplo: 1/10 + 1/100 + 
>> >1/1000 
>> > > + ... . Bom e agora, o que tudo isto tem a ver com sua pergunta? 
>> > > 
>> > > Espero ter ajudado, apesar dessa resposta meio enigmática, mas acho 
>que 
>> > > assim auxilio mais! 
>> > > 
>> > > Frederico. 
>> > > 
>> > > >From: "Marcelo Augusto Pereira" 
>> > > >Reply-To: [EMAIL PROTECTED] 
>> > > >To: 
>> > > >Subject: [obm-l] Impossibilidade do movimento 
>> > > >Date: Fri, 23 Jan 2004 19:05:25 -0200 
>> > > > 
>> > > >Entre dois números reais há infinitos outros. Considere um segmento 
>de 
>> >reta 
>> > > >com o número 0 assinalado em uma ponta e o número 1 marcado na 
outra. 
>> > > >Considere também que esse segmento de reta foi representado no chão 
>com 
>> >um 
>> > > >risco de um metro de comprimento. Para cada número entre 0 e 1 há um 
>> >ponto 
>> > > >correspondente no segmento de reta e, conseqüentemente, no risco 
>> >marcado 
>> >no 
>> > > >chão. Como eu consigo caminhar do ponto 0 até o ponto 1, se para 
>chegar 
>> >de 
>> > > >0 
>> > > >até 1 eu tenho que passar por infinitos pontos? 
>> > > > 
>> > > 
>> > 
>>= 
>> > > >Instruções para entrar na lista, sair da lista e usar a lista em 
>> > > >http://www.mat.puc-rio.br/~nicolau/olimp/obm-l.html 
>> > > 
>> > 
>>= 
>> > > 
>> > > _ 
>> > > MSN Hotmail, o maior webmail do Brasil. http://www.hotmail.com 
>> > > 
>> > > 
>> 
>= 
>> > > Instruções para entrar na lista, sair da lista e usar a lista e

Re: [obm-l] CN-97_

2004-01-21 Por tôpico Fabio Henrique
Se escrever 147 algarismos você terá 31707 vinte e nove vezes e, a seguir, 3 
e 1 que somam 526. Para exceder de 530 é necessário o próximo dígito: 7. 

Resp: E 

Em 20 Jan 2004, [EMAIL PROTECTED] escreveu: 

>> Um aluno, efetuando a divisão de 13 por 41, foi 
>> determinando o quociente até a soma de todos os 
>> algarismos por ele escritos, na parte decimal, foi 
>> imediatamente maior ou igual a 530. Quantas casas 
>> decimais ele escreveu? 
>> 
>> (A) 144 (D) 147 
>> (B) 145 (E) 148 
>> (C) 146 
> 
>13/41 = 0.31707317073... 
> 
>É uma dízima períodica. 
> 
>Temos que 3 + 1 + 7 + 0 + 7 = 18 
>Então temos 530/18 = 29.44... 
> 
>Então, 5*29.4... = 147.22 
> 
>Achei letra (D). 
> 
>Abraço, 
>Henrique. 
> 
>= 
>Instruções para entrar na lista, sair da lista e usar a lista em 
>http://www.mat.puc-rio.br/~nicolau/olimp/obm-l.html 
>= 
> 
>-- 

_
Voce quer um iGMail protegido contra vírus e spams? 
Clique aqui: http://www.igmailseguro.ig.com.br



Re: [obm-l] OBM:LISTA MONTE DE MERDA

2004-01-04 Por tôpico Fabio Henrique
Olha, eu realmente gostaria de saber o motivo de tanta raiva. Eu sei que 
talvez jamais saiba porque insiste em provocar os participantes desta lista, 
porque este ódio para com o Nicolau e porque perde tempo com isso. 
Por que manter este comportamento psicótico e previsível. Agora, você vai 
dirigir os seus xingamentos a mim. Tudo isso faz parte deste comportamento 
típico de pessoas que não conseguem lidar com dificuldades mantendo o 
equilíbrio. 
Cara, já deu pra ver que você é bom pra cacete quando se trata de 
computadores. Se eu soubesse metade do que você sabe sobre o assunto, 
certamente seria o fundador e moderador de uma lista sobre isto. Mostraria a 
parte prática do conhecimento. 
Talvez, você ainda seja egoísta o bastante para tomar este tipo de 
iniciativa. 
E agora? Você vai me bombardear? É possível e muito provável. No entanto, se 
você pelo menos parar para considerar o que escrevo, numa boa, sem este 
desdém costumeiro, quem sabe muitas pessoas consigam crescer graças a um 
movimento seu, quem sabe, daqui a alguns anos, muitos lhe sejam gratos. 
Fabio. 


Em  1 Jan 2004, [EMAIL PROTECTED] escreveu: 

>MagazinesNewspapersNewslettersAll Products 
>More Search Options 
> 
>Professional & Trade 
>Education 
>Engineering 
>Medicine 
> See all 
> 
>- 
>--- 
>Browse by Subject 
>Arts & Crafts 
>Automotive 
>Bridal 
>Business & Finance 
>Children's 
>Computer & Internet 
>Electronics & Audio 
>Entertainment 
>Family & Parenting 
>Fashion & Style 
>Food & Gourmet 
>Games & Hobbies 
>Gay & Lesbian 
>Health & Fitness 
>History 
>Home & Garden 
>International 
>Lifestyle & Cultures 
>Literary 
>Men's Interest 
>Music 
>News & Politics 
>Newspapers 
>Pets 
>Professional & Trade 
>Religion & Spirituality 
>Science & Nature 
>Spanish-Language 
>Sports & Leisure 
>Teens 
>Travel & Regional 
>Women's Interest 
> 
>Need Help? 
> 
>Magazine Subscriptions FAQ 
>Help Desk 
>Publisher Services 
>Send Feedback 
> 
>Health & Fitness Magazines 
> Inspire your mind and body--choose from over 200 
>Health & Fitness magazines, including Men's Health, 
>Shape, and Self. 
> 
>Subscribe and Save 
> Get $5 to $20 off a future Amazon.com purchase when 
>you subscribe to one of our select magazine titles-- 
>including People, Wired, and Sports Illustrated. 
> 
>Classic Magazines 
> If you're a devoted subscriber to National Geographic, 
>we recommend trying one of these other quality 
>magazines: National Geographic Kids, National Geographic 
>Traveler, or National Geographic Adventure. 
> 
>Movers and Shakers 
>Scientific American 
> 766% today ~ Amazon.com Sales Rank: 6 (was 52) 
>From Amazon.com 
>For working scientists, especially in high-tech fields, 
>there are only a few crucial nonjournal periodicals to 
>pore over faithfully, and Scientific American is one 
>of... Read more 
> 
> 800% Real Simple [MAGAZINE SUBSCRIPTION] Magazine 
>Subscription 
> 787% Southern Living [MAGAZINE SUBSCRIPTION] Magazine 
>Subscription 
> 783% Hot Rod [MAGAZINE SUBSCRIPTION] Magazine 
>Subscription 
> 
> More Movers & Shakers 
> 
>Your Lifestyle 
>Water Sports: Make Waves 
> 
>Boating Life [MAGAZINE SUBSCRIPTION] 
>Magazine Subscription 
>Alkaline AA 12-Pack Batteries 
>by Rayovac 
> 
>More in Water Sports: Make Waves 
> Deal of the Day 
> Subscribe to the new music magazine Tracks and get $15 
>off a future Amazon.com purchase. 
> 
>Subscriptions for $10 or Less 
> Stuff their stocking with a magazine subscription for 
>$10 or less, including Good Housekeeping, Esquire, and 
>Parents. 
> 
>Top Selling Subscriptions 
>Our Bestsellers, Updated Hourly 
> 
>InStyle [MAGAZINE SUBSCRIPTION] Magazine Subscription 
>O : The Oprah Magazine [MAGAZINE SUBSCRIPTION] Magazine 
>Subscription 
>Real Simple [MAGAZINE SUBSCRIPTION] Magazine 
>Subscription 
>Sports Illustrated [MAGAZINE SUBSCRIPTION] Magazine 
>Subscription 
>Smithsonian [MAGAZINE SUBSCRIPTION] Magazine 
>Subscription 
>Scientific American [MAGAZINE SUBSCRIPTION] Magazine 
>Subscription 
>Wired [MAGAZINE SUBSCRIPTION] Magazine Subscription 
>ESPN the Magazine [MAGAZINE SUBSCRIPTION] Magazine 
>Subscription 
> More Top Sellers 
> 
>Top Selling Subscriptions in Business & Finance 
>SmartMoney [MAGAZINE SUBSCRIPTION] Magazine Subscription 
>$12.00 
>Consumer Reports [MAGAZINE SUBSCRIPTION] Magazine 
>Subscription $26.00 
>The Economist [MAGAZINE SUBSCRIPTION] Magazine 
>Subscription $129.00 
> More Top Selling Subscriptions in Business & Finance 
> 
>- 
>--- 
>More to Explore 
>E-mail Recommendations 
>Learn about Co-op Build Your Own Online Magazine Store 
> Team up with Amazon.com to sell magazines from your Web 
>site. We'll help you. Join Amazon.com Associates today! 
>Sell Your Used Items 
> 
>__ 
>Acabe com aquelas janelinhas que pulam na sua tela. 
>AntiPop-u

Re: [obm-l] erro no mathematica?

2003-12-10 Por tôpico Fabio Henrique
Alguém pode me esclarecer o que é este mathematica? Onde consigo. Desculpem 
a ignorância... 


Em 10 Dec 2003, [EMAIL PROTECTED] escreveu: 

>-- 
> 
> No mathematica 4, a resposta é: 
> 
> -pi/2 +( 3Sqrt[3] + 2pi)/3 = Sqrt[3] + pi/6 
> 
> ---Original Message--- 
> 
> From: [EMAIL PROTECTED] 
> Date: quarta-feira, 10 de dezembro de 2003 10:35:38 
> To: [EMAIL PROTECTED] 
> Subject: Re: [obm-l] erro no mathematica? 
> 
> > Integral de 0 até pi de Abs[1/2 + Cos[t]] dt 
> > ele me devolve -pi/2 ? 
> 
> a integral de uma função f(x)>= 0 no intervalo de 
> integração NUNCA será um número negativo. Certamente é 
> um erro do Mathematica. 
> 
> __ 
> Do you Yahoo!? 
> New Yahoo! Photos - easier uploading and sharing. 
> http://photos.yahoo.com/ 
> = 
> Instruções para entrar na lista, sair da lista e usar a lista em 
> http://www.mat.puc-rio.br/~nicolau/olimp/obm-l.html 
> = 
> . 
> 
>  
> IncrediMail - Email has finally evolved - 
> 
>-- 

_
Voce quer um iGMail protegido contra vírus e spams?
Clique aqui: http://www.igmailseguro.ig.com.br
Ofertas imperdíveis! Link: http://www.americanas.com.br/ig/
Ofertas imperdíveis!

=
Instruções para entrar na lista, sair da lista e usar a lista em
http://www.mat.puc-rio.br/~nicolau/olimp/obm-l.html
=


Re: [obm-l] erro no mathematica?

2003-12-10 Por tôpico Fabio Henrique
Alguém pode me esclarecer o que é este mathematica? Onde consigo. Desculpem 
a ignorância... 


Em 10 Dec 2003, [EMAIL PROTECTED] escreveu: 

>-- 
> 
> No mathematica 4, a resposta é: 
> 
> -pi/2 +( 3Sqrt[3] + 2pi)/3 = Sqrt[3] + pi/6 
> 
> ---Original Message--- 
> 
> From: [EMAIL PROTECTED] 
> Date: quarta-feira, 10 de dezembro de 2003 10:35:38 
> To: [EMAIL PROTECTED] 
> Subject: Re: [obm-l] erro no mathematica? 
> 
> > Integral de 0 até pi de Abs[1/2 + Cos[t]] dt 
> > ele me devolve -pi/2 ? 
> 
> a integral de uma função f(x)>= 0 no intervalo de 
> integração NUNCA será um número negativo. Certamente é 
> um erro do Mathematica. 
> 
> __ 
> Do you Yahoo!? 
> New Yahoo! Photos - easier uploading and sharing. 
> http://photos.yahoo.com/ 
> = 
> Instruções para entrar na lista, sair da lista e usar a lista em 
> http://www.mat.puc-rio.br/~nicolau/olimp/obm-l.html 
> = 
> . 
> 
>  
> IncrediMail - Email has finally evolved - 
> 
>-- 

_
Voce quer um iGMail protegido contra vírus e spams?
Clique aqui: http://www.igmailseguro.ig.com.br
Ofertas imperdíveis! Link: http://www.americanas.com.br/ig/
Ofertas imperdíveis!

=
Instruções para entrar na lista, sair da lista e usar a lista em
http://www.mat.puc-rio.br/~nicolau/olimp/obm-l.html
=


Re: [obm-l] erro no mathematica?

2003-12-10 Por tôpico Fabio Henrique
Obrigado pelo esclarecimento mas o comentário me ofendeu.Em 10 Dec 2003, 
[EMAIL PROTECTED] escreveu: 

>On Wed, Dec 10, 2003 at 06:53:36AM -0200, Fabio Henrique wrote: 
>> Como consigo o mathematica? 
> 
>É um software comercial, para tê-lo legalmente você precisa comprar. 
>A home page do produto é 
>http://www.wolfram.com/products/mathematica/index.html 
> 
>E por favor, nada de discutir pirataria aqui. 
> 
>[]s, N. 
>= 
>Instruções para entrar na lista, sair da lista e usar a lista em 
>http://www.mat.puc-rio.br/~nicolau/olimp/obm-l.html 
>= 
> 
>-- 

_
Voce quer um iGMail protegido contra vírus e spams? 
Clique aqui: http://www.igmailseguro.ig.com.br
Ofertas imperdíveis! Link: http://www.americanas.com.br/ig/
Ofertas imperdíveis!

=
Instruções para entrar na lista, sair da lista e usar a lista em
http://www.mat.puc-rio.br/~nicolau/olimp/obm-l.html
=


Re: [obm-l] erro no mathematica?

2003-12-10 Por tôpico Fabio Henrique
Como consigo o mathematica? 


Em 09 Dec 2003, [EMAIL PROTECTED] escreveu: 

>pessoal, pq no mathematica (5.0) quando eu calculo 
> 
>Integral de 0 até pi de Abs[1/2 + Cos[t]] dt 
>ele me devolve -pi/2 ? 
>a resposta nao é sqrt(3) + pi/6 ? 
> 
>obrigado! 
> 
>= 
>Instruções para entrar na lista, sair da lista e usar a lista em 
>http://www.mat.puc-rio.br/~nicolau/olimp/obm-l.html 
>= 
> 
>-- 

_
Voce quer um iGMail protegido contra vírus e spams? 
Clique aqui: http://www.igmailseguro.ig.com.br
Ofertas imperdíveis! Link: http://www.americanas.com.br/ig/
Ofertas imperdíveis!

=
Instruções para entrar na lista, sair da lista e usar a lista em
http://www.mat.puc-rio.br/~nicolau/olimp/obm-l.html
=


[obm-l] Re: Equação

2003-12-04 Por tôpico Fabio Henrique
Pedro, observe que o primeiro módulo zera para x = 1 e o segundo para x = 
-5. Vamos então dividir a nossa reta real em 3 pedaços: MENOR QUE OU IGUAL A 
-5, ENTRE -5 E 1 e MAIOR QUE OU IGUAL A 1. 

Na primeira região, trocaremos mod(x-1) por -x+1  e  mod(x+5) por -x-5. 
Assim ficaremos com: 

-2x-4=6 => x=-5. Este resultado está no intervalo em que estamos 
trabalhando, portanto é válido. 

Na segunda região, trocaremos mod(x-1) por -x+1 e mod(x+5) por x+5. Assim 
ficaremos com: 

x+5-x+1=6 => 6=6. Isto quer dizer que todos os valores deste intervalo são 
soluções da tua equação. 

Na terceira região, trocaremos mod(x-1) por x-1 e mod(x+5) por x+5. Assim 
ficaremos com: 

2x+4=6 => x=1. Este resultado está no intervalo em que estamos trabalhando, 
portanto é válido. 

Assim, a resposta é [-5,1] 



Em 4 Dec 2003, [EMAIL PROTECTED] escreveu: 

>oi, pessoal 
> 
> Não conseguir 
>resolver estas questões de um concurso , ajude-me 
> 
> 1) A soma dos valores inteiros positivos e 
>negativos de x que satisfazem a equação: 
> 
>l x - 1 l + l x + 5 l = 6 
> ( equação modular ) 
> 
> a) 
>-14 
> 
>b) -13 
> c) -12 d) 
>-11 
> 
> 2) Usando uma vez a letra X , uma vez a letra 
>Y e N - 2 vezes a letra Z , podemos formar 20 anagramas diferentes 
>com n letras em cada anagrama. O valor de n^2 é : 
> 
> a) 
>9 
>b)16 
>c)25 d) 36 
> 
> 3) ma motocicleta cujo preço de tabela é R$ 
>8.000,00 é vendida , à vista , com desconto de x% ou em duas 
>parcelas iguais 
> de R$ 4.000,00 , sendo a 
>primeira no ato da compra e segunda um mês após a compra. Supondo 
>que o Sr. Paulo dispõe 
> do dinheiro necessário 
>para pagar à vista e que ele sabe que a diferença entre o preço à vista e 
>a primeira parcela pode ser 
> aplicada no mercado financeiro a 
>uma taxa de 25% ao mês. Nessas condições , qual é o valor de x que torna 
>indiferente 
> comprar à vista ou a prazo 
>: 
> 
> a) 
>10 
>b)12 c) 
>15 D) 16 
> 
> -- 
> Esta mensagem foi verificada pelo sistema de antivírus e 
> acredita-se estar livre de perigo. 
> 
>-- 

_
Voce quer um iGMail protegido contra vírus e spams? 
Clique aqui: http://www.igmailseguro.ig.com.br
Ofertas imperdíveis! Link: http://www.americanas.com.br/ig/
Ofertas imperdíveis!

=
Instruções para entrar na lista, sair da lista e usar a lista em
http://www.mat.puc-rio.br/~nicolau/olimp/obm-l.html
=


Re: [obm-l] equação!!

2003-11-09 Por tôpico Fabio Henrique
Observe que f(4)=-387 e f(5)=53. Então existe uma raiz entre 4 e 5. Sugiro 
utilizar o método de Newton-Raphson 
Abraços. Fabio Henrique. 

Em 28 Oct 2003, [EMAIL PROTECTED] escreveu: 

>como posso resolver a seguinte equação? 
> 
> (x^4) + (x^3) + (x^2) + (x) = 727 
> 
> (consegui fazer na máquina, pois ainda naum tive muito tempo para 
resolvê-la algebricamente). 
> 
>-- 

_
Voce quer um iGMail protegido contra vírus e spams? 
Clique aqui: http://www.igmailseguro.ig.com.br
Ofertas imperdíveis! Link: http://www.americanas.com.br/ig/
Ofertas imperdíveis!

=
Instruções para entrar na lista, sair da lista e usar a lista em
http://www.mat.puc-rio.br/~nicolau/olimp/obm-l.html
=


Re: [obm-l] a^3+b^3+c^3 = 3abc

2003-11-09 Por tôpico Fabio Henrique
Pessoal, consegui fazendo (a+b+c)^3. 
Isto vai dar a^3 + b^3 + c^3 + 3ba^2 + 3ab^2 + 3ca^2 + 3ac^2 + 3bc^2 + 3cb^2 
+ 6abc = 0. 

a^3 + b^3 + c^3 + 3ab(a+b) + 3ac(a+c) + 3cb(b+c) + 6abc = 0. 

a^3 + b^3 + c^3 + 3ab(-c) + 3ac(-b) + 3cb(-a) + 6abc = 0. 

Abraços. Fabio Henrique. 



>Gostaria de tentar uma resoluçao sobre o enunciado, só que fazendo um 
>caminho inverso: 
> 
>Dado a+b+c=0, 
>quero chegar em 
> 
>a^3 + b^3 + c^3 - 3abc = 0. 
> 
>Partindo de: 
> 
>a^3 + b^3 + c^3 - 3abc 
> 
>Farei a linha acima por determinante: 
> 
> a b c 
> c a b 
> b c a 
> 
>A soma de cada linha deste deteminante eh a+b+c que como jah eh sabido eh 
>zero. 
> 
>logo o determinante acima eh igual a zero. 
> 
>Assim temos: 
> 
>a^3 + b^3 + c^3 - 3abc = 0 
> 
>e 
> 
>a^3 + b^3 + c^3 = 3abc 
> 
>Por favor me corrijam se eu estiver errado. 
> 
>Obrigado. 
> 
>>-- Mensagem original -- 
>> 
>> >Ola pessoal, 
>> > 
>> >Depois de alguns meses afastado da lista e sem estudar matematica, pois 
>> 
>> >estava estudando para um concurso e acabei de faze-lo. Agora eh esperar 
>>ansioso 
>> > 
>> >pelo resultado que sairah em menos de 2 semanas. Para nao ficar 
off-topic 
>> >vou 
>> >re-comecar a postar minhas duvidas. Vamos la: 
>> > 
>> >1) Prove que se a + b + c = 0, entao a^3 + b^3 + c^3 = 3abc 
>> > 
>> >Obs: Como estou voltando agora, desculpem me se o problema for trivial. 
>> 
>> >Preciso me desenferrujar aos poucos ;-) em matematica e pegar o ritmo de 
>> >novo. 
>> > 
>> > 
>> > 
>> 
>> 
>> 
>>-- 
>>Use o melhor sistema de busca da Internet 
>>Radar UOL - http://www.radaruol.com.br 
>> 
>> 
>> 
>>>Instruções 
>para entrar na lista, sair da lista e usar a lista em 
>>http://www.mat.puc-rio.br/~nicolau/olimp/obm-l.html 
>> 
>_ 
>MSN Hotmail, o maior webmail do Brasil. http://www.hotmail.com 
> 
>Instruções 
>para entrar na lista, sair da lista e usar a lista em 
>http://www.mat.puc-rio.br/~nicolau/olimp/obm-l.html 
> 
> 
>-- 

_
Voce quer um iGMail protegido contra vírus e spams? 
Clique aqui: http://www.igmailseguro.ig.com.br
Ofertas imperdíveis! Link: http://www.americanas.com.br/ig/
Ofertas imperdíveis!

=
Instruções para entrar na lista, sair da lista e usar a lista em
http://www.mat.puc-rio.br/~nicolau/olimp/obm-l.html
=


Re: [obm-l] Re: N/A correçao

2003-11-08 Por tôpico Fabio Henrique
Pessoal, resolvi usando o 1º lema de Kaplansky. Desta forma, calculei 
quantos são sem zeros, com exatamente 1 zero, com exatamente 2 zeros, com 
exatamente 3 zeros e com exatamente 4 zeros. Encontrei, respectivamente, os 
valores 1, 8, 21, 20 e 5. Somados dão 55. 

Abraços, Fabio Henrique. 



 Em 4 Nov 2003, [EMAIL PROTECTED] escreveu: 

>Conforme o Stabel ja apontou, ha um erro de soma abaixo. 
>f(7) = 21 + 13 = 34 e f(8) = 34+21 = 55. 
> 
>-- Original Message --- 
>From: "Augusto Cesar de Oliveira Morgado" 
>To: [EMAIL PROTECTED] 
>Sent: Mon, 3 Nov 2003 20:59:29 -0200 
>Subject: [obm-l] Re: N/A 
> 
>> Seja f(n) a resposta para uma sequencia de n bits. Ou a seq. começa 
>> em 1 ou começa em 01. Logo, f(n)=f(n-1)+f(n-2). Como f(1) = 2 e f(2) 
>> = 3, f(3) = 2+3=5, f(4) = 5+3 = 8, f(5) = 8+5 = 13, f(6)=8 = 21, 
>> f(7) = 21+13 = 44 e f(8) = 44+21 = 65. 
>> 
>> -- 
>> CIP WebMAIL - Nova Geração - v. 2.1 
>> CentroIn Internet Provider http://www.centroin.com.br 
>> Tel: (21) 2542-4849, (21) 2295-3331 Fax: (21) 2295-2978 
>> Empresa 100% Brasileira - Desde 1992 
>> 
>> -- Original Message --- 
>> From: "Daniel Faria" 
>> To: [EMAIL PROTECTED] 
>> Sent: Mon, 03 Nov 2003 19:16:55 -0200 
>> Subject: N/A 
>> 
>> > Ainda nao consegui finalizar este exercício: 
>> > 
>> > De quantas maneiras podemos formar uma sequencia de oito bits(0 ou 1) 
>> > de forma que nunca apareça nesta sequencia zeros adjacentes ( _ _ 
>> > 0 0 _ _ _ _ ). 
>> > 
>> > Obrigado. 
>> > 
>> > _ 
>> > MSN Hotmail, o maior webmail do Brasil. http://www.hotmail.com 
>> > 
>> > 
>> > 
>Instruções para entrar na lista, sair da lista e usar a lista em 
>> > http://www.mat.puc-rio.br/~nicolau/olimp/obm-l.html 
>> > 
>> 
>--- End of Original Message --- 
>> 
>> > 
>Instruções para entrar na lista, sair da lista e usar a lista em 
>> http://www.mat.puc-rio.br/~nicolau/olimp/obm-l.html 
>> 
>--- 
>End of Original Message --- 
> 
>Instruções 
>para entrar na lista, sair da lista e usar a lista em 
>http://www.mat.puc-rio.br/~nicolau/olimp/obm-l.html 
> 
> 
>-- 

_
Voce quer um iGMail protegido contra vírus e spams? 
Clique aqui: http://www.igmailseguro.ig.com.br
Ofertas imperdíveis! Link: http://www.americanas.com.br/ig/
Ofertas imperdíveis!

=
Instruções para entrar na lista, sair da lista e usar a lista em
http://www.mat.puc-rio.br/~nicolau/olimp/obm-l.html
=


Re: [obm-l] Re: N/A correçao

2003-11-08 Por tôpico Fabio Henrique
Pessoal, resolvi usando o 1º lema de Kaplansky. Desta forma, calculei 
quantos são sem zeros, com exatamente 1 zero, com exatamente 2 zeros, com 
exatamente 3 zeros e com exatamente 4 zeros. Encontrei, respectivamente, os 
valores 1, 8, 21, 20 e 5. Somados dão 55. 

Abraços, Fabio Henrique. 



 Em 4 Nov 2003, [EMAIL PROTECTED] escreveu: 

>Conforme o Stabel ja apontou, ha um erro de soma abaixo. 
>f(7) = 21 + 13 = 34 e f(8) = 34+21 = 55. 
> 
>-- Original Message --- 
>From: "Augusto Cesar de Oliveira Morgado" 
>To: [EMAIL PROTECTED] 
>Sent: Mon, 3 Nov 2003 20:59:29 -0200 
>Subject: [obm-l] Re: N/A 
> 
>> Seja f(n) a resposta para uma sequencia de n bits. Ou a seq. começa 
>> em 1 ou começa em 01. Logo, f(n)=f(n-1)+f(n-2). Como f(1) = 2 e f(2) 
>> = 3, f(3) = 2+3=5, f(4) = 5+3 = 8, f(5) = 8+5 = 13, f(6)=8 = 21, 
>> f(7) = 21+13 = 44 e f(8) = 44+21 = 65. 
>> 
>> -- 
>> CIP WebMAIL - Nova Geração - v. 2.1 
>> CentroIn Internet Provider http://www.centroin.com.br 
>> Tel: (21) 2542-4849, (21) 2295-3331 Fax: (21) 2295-2978 
>> Empresa 100% Brasileira - Desde 1992 
>> 
>> -- Original Message --- 
>> From: "Daniel Faria" 
>> To: [EMAIL PROTECTED] 
>> Sent: Mon, 03 Nov 2003 19:16:55 -0200 
>> Subject: N/A 
>> 
>> > Ainda nao consegui finalizar este exercício: 
>> > 
>> > De quantas maneiras podemos formar uma sequencia de oito bits(0 ou 1) 
>> > de forma que nunca apareça nesta sequencia zeros adjacentes ( _ _ 
>> > 0 0 _ _ _ _ ). 
>> > 
>> > Obrigado. 
>> > 
>> > _ 
>> > MSN Hotmail, o maior webmail do Brasil. http://www.hotmail.com 
>> > 
>> > 
>> > 
>Instruções para entrar na lista, sair da lista e usar a lista em 
>> > http://www.mat.puc-rio.br/~nicolau/olimp/obm-l.html 
>> > 
>> 
>--- End of Original Message --- 
>> 
>> > 
>Instruções para entrar na lista, sair da lista e usar a lista em 
>> http://www.mat.puc-rio.br/~nicolau/olimp/obm-l.html 
>> 
>--- 
>End of Original Message --- 
> 
>Instruções 
>para entrar na lista, sair da lista e usar a lista em 
>http://www.mat.puc-rio.br/~nicolau/olimp/obm-l.html 
> 
> 
>-- 

_
Voce quer um iGMail protegido contra vírus e spams? 
Clique aqui: http://www.igmailseguro.ig.com.br
Ofertas imperdíveis! Link: http://www.americanas.com.br/ig/
Ofertas imperdíveis!

=
Instruções para entrar na lista, sair da lista e usar a lista em
http://www.mat.puc-rio.br/~nicolau/olimp/obm-l.html
=


Re: [obm-l] <

2003-10-17 Por tôpico Fabio Henrique
Nicolau, sou só eu que estou recebendo 50 mensagens iguais a esta por dia? o 
que está havendo? 

Abraços. Fabio. 


Em 16 Oct 2003, [EMAIL PROTECTED] escreveu: 

>= 
>Instruções para entrar na lista, sair da lista e usar a lista em 
>http://www.mat.puc-rio.br/~nicolau/olimp/obm-l.html 
>= 
> 
>-- 

_
Voce quer um iGMail protegido contra vírus e spams?
Clique aqui: http://www.igmailseguro.ig.com.br
Ofertas imperdíveis! Link: http://www.americanas.com.br/ig/
Ofertas imperdíveis!

=
Instruções para entrar na lista, sair da lista e usar a lista em
http://www.mat.puc-rio.br/~nicolau/olimp/obm-l.html
=


Re: [obm-l] Triângulos_(Mr._Crowley)

2003-09-11 Por tôpico Fabio Henrique
Estava lendo mensagens antigas e acho que encontrei um erro. O lado AB vale 
2sqrt(13). 
Fabio. 





Em 4 Sep 2003, [EMAIL PROTECTED] escreveu: 

>Legal cara,ce e o mesmo que foi homenageado pelo 
>Ozzy Osbourne ou colocou este nick como eu fiz o 
>meu? 
> --- Andre Araujo escreveu: 
>> 
>> > 
>> >AB é a hipotenusa de um triângulo retângulo 
>> ABC. A 
>> >mediana AD mede 7 e a mediana BE mede 4. O 
>> comprimento 
>> >AB é igual a: 
>> 
>> Pitagoras no triangulo BCE: BC^2+(AC/2)^2=BE^2 
>> Pitagoras no triangulo ACD: (BC/2)^2+AC^2=AD^2 
>> 
>> Somando as duas equacoes, temos: 
>> 
>> (5/4)*(BC^2+AC^2)=16+49, mas BC^2+AC^2=AB^2. 
>> Logo: 
>> 
>> AB^2=4*65/5=>AB=2sqrt(15). 
>> 
>> > 
>> >a)2·sqrt(3) 
>> >b)5·sqrt(2) 
>> >c)5·sqrt(3) 
>> >d)10 
>> >e)n.d.a 
>> > 
>> >ABC é um triângulo e M é um ponto médio sobre 
>> o lado 
>> >BC, tal que MC=2MB. A razão entre as área dos 
>> triângulos 
>> >ABC e MAC é: 
>> 
>> Note que a altura relativa ao lado BC(h)do 
>> triangulo ABC eh igual a altura 
>> relativa ao lado MC do triangulo AMC. Logo: 
>> 
>> S(ABC)=BC*h/2 
>> S(AMC)=MC*h/2 
>> 
>> S(ABC)/S(AMC)=BC/MC=(MC+MB)/MC=(2*MB+MB)/2*MB = 
>> 3/2. 
>> 
>> > 
>> >a)4 b)3 c)2 d)9/4 e)3/2 
>> > 
>> >Grato 
>> > 
>> >Mr. Crowley 
>> > 
>> 
>>__ 
>> 
>> >Acabe com aquelas janelinhas que pulam na sua 
>> tela. 
>> >AntiPop-up UOL - É grátis! 
>> >http://antipopup.uol.com.br/ 
>> > 
>> 
>>= 
>> 
>> >Instruções para entrar na lista, sair da lista 
>> e usar a lista em 
>> 
>>http://www.mat.puc-rio.br/~nicolau/olimp/obm-l.html 
>> 
>> 
>>= 
>> 
>> > 
>> >-- 
>> 
>> 
>_ 
>> Voce quer um iGMail protegido contra vírus e 
>> spams? 
>> Clique aqui: http://www.igmailseguro.ig.com.br 
>> Ofertas imperdíveis! Link: 
>> http://www.americanas.com.br/ig/ 
>> 
>> 
>= 
>> Instruções para entrar na lista, sair da lista 
>> e usar a lista em 
>> 
>http://www.mat.puc-rio.br/~nicolau/olimp/obm-l.html 
>> 
>= 
> 
>___ 
>Desafio AntiZona: participe do jogo de perguntas e respostas que vai 
>dar um Renault Clio, computadores, câmeras digitais, videogames e muito 
>mais! www.cade.com.br/antizona 
>= 
>Instruções para entrar na lista, sair da lista e usar a lista em 
>http://www.mat.puc-rio.br/~nicolau/olimp/obm-l.html 
>= 
> 
>-- 

_
Voce quer um iGMail protegido contra vírus e spams? 
Clique aqui: http://www.igmailseguro.ig.com.br
Ofertas imperdíveis! Link: http://www.americanas.com.br/ig/
Ofertas imperdíveis!

=
Instruções para entrar na lista, sair da lista e usar a lista em
http://www.mat.puc-rio.br/~nicolau/olimp/obm-l.html
=


Re: [obm-l] CN - 2001_AJUDEM-ME

2003-08-31 Por tôpico Fabio Henrique
Se o enunciado está corretamente transcrito, você já sabe que z = 1. 
Daí fica fácil... 


Encontrei x=90 e y=11 

Em 31 Aug 2003, [EMAIL PROTECTED] escreveu: 

>Marta comprou petecas, bolas e boneca, pagando por 
> cada unidade, respectivamente, 1,00; 10,00; 20,00. 
> Gastou 220,00 em um total de 102 unidades desses 
> brinquedos. Quantas petecas ela comprou? 
> 
>Seja x o número de petecas 
> y o número de bolas 
> z o número de bonecas 
>equacionando devidamente, temos: 
>1x+10y+20z=220 
>x+y+z=102 
> 
>olá pessoal, como faço para resolver esta quetão, já q 
>nao consigo resolver esta equação? 
> 
>___ 
>Desafio AntiZona: participe do jogo de perguntas e respostas que vai 
>dar um Renault Clio, computadores, câmeras digitais, videogames e muito 
>mais! www.cade.com.br/antizona 
>= 
>Instruções para entrar na lista, sair da lista e usar a lista em 
>http://www.mat.puc-rio.br/~nicolau/olimp/obm-l.html 
>= 
> 
>-- 

_
Voce quer um iGMail protegido contra vírus e spams? 
Clique aqui: http://www.igmailseguro.ig.com.br
Ofertas imperdíveis! Link: http://www.americanas.com.br/ig/
Ofertas imperdíveis!

=
Instruções para entrar na lista, sair da lista e usar a lista em
http://www.mat.puc-rio.br/~nicolau/olimp/obm-l.html
=


  1   2   >